Jump to content

Wikipedia:Reference desk/Science: Difference between revisions

From Wikipedia, the free encyclopedia
Content deleted Content added
Line 577: Line 577:
:::Try search "bicycle crank mechanism" - many results many of which are patents. Here's one example [http://wiki.anyscript.org/images/9/9d/Riofullpaper.pdf] [[Special:Contributions/87.102.17.246|87.102.17.246]] ([[User talk:87.102.17.246|talk]]) 08:52, 8 June 2010 (UTC)
:::Try search "bicycle crank mechanism" - many results many of which are patents. Here's one example [http://wiki.anyscript.org/images/9/9d/Riofullpaper.pdf] [[Special:Contributions/87.102.17.246|87.102.17.246]] ([[User talk:87.102.17.246|talk]]) 08:52, 8 June 2010 (UTC)
:::On a similar line elliptic and ovoid drives have been produced eg search "elliptical bicycle crank"
:::On a similar line elliptic and ovoid drives have been produced eg search "elliptical bicycle crank"
:::I've no idea if any of these are any good or just 'bright ideas'.[[Special:Contributions/87.102.17.246|87.102.17.246]] ([[User talk:87.102.17.246|talk]]) 09:08, 8 June 2010 (UTC)
:::I've no idea if any of these are any good or just 'bright ideas'. These are mostly designs to eliminate dead spots, haven't found a straight stroke mechanism yet. But they definately exist - no problem buying them. [[Special:Contributions/87.102.17.246|87.102.17.246]] ([[User talk:87.102.17.246|talk]]) 09:08, 8 June 2010 (UTC)


== how does water eat thru polyurethane thou? ==
== how does water eat thru polyurethane thou? ==

Revision as of 09:18, 8 June 2010

Welcome to the science section
of the Wikipedia reference desk.
Select a section:
Want a faster answer?

Main page: Help searching Wikipedia

   

How can I get my question answered?

  • Select the section of the desk that best fits the general topic of your question (see the navigation column to the right).
  • Post your question to only one section, providing a short header that gives the topic of your question.
  • Type '~~~~' (that is, four tilde characters) at the end – this signs and dates your contribution so we know who wrote what and when.
  • Don't post personal contact information – it will be removed. Any answers will be provided here.
  • Please be as specific as possible, and include all relevant context – the usefulness of answers may depend on the context.
  • Note:
    • We don't answer (and may remove) questions that require medical diagnosis or legal advice.
    • We don't answer requests for opinions, predictions or debate.
    • We don't do your homework for you, though we'll help you past the stuck point.
    • We don't conduct original research or provide a free source of ideas, but we'll help you find information you need.



How do I answer a question?

Main page: Wikipedia:Reference desk/Guidelines

  • The best answers address the question directly, and back up facts with wikilinks and links to sources. Do not edit others' comments and do not give any medical or legal advice.
See also:



June 4

How does a battery-less moisture-meter work?

How would a moisture meter such as this work, bearing in mind that it uses no batteries? When inserting into soil, the needle flicks to the right before slowly moving a distance to the left and settling. Which of the readings can be considered the proper moisture level of the soil, based on how the device supposedly works? Would it be likely to harm the meter, to leave it sitting in soil, rather than removing and cleaning it every time? --92.25.237.15 (talk) 01:18, 4 June 2010 (UTC)[reply]

The picture clearly shows the two electrodes are made of different metals, so I would imagine it works in a similar way to a lemon battery. In the normal use of the device, the electrodes will be eroded (that's how it works - see the article I linked to), so leaving it in the soil will drastically reduce its working lifetime. --Tango (talk) 01:34, 4 June 2010 (UTC)[reply]
Have you tried covering the light sensor of the light meter? If the device stops working it might mean it uses the light as a source of energy. Either way, leaving it in the soil might not be a good idea. Have you tried asking the manufacturer? Dauto (talk) 02:45, 4 June 2010 (UTC)[reply]
That's a quote from the link provided by the OP "To avoid damaging the electrode, please clean the electrode after each use". I guess that answers the question. Dauto (talk) 02:50, 4 June 2010 (UTC)[reply]

Does Synthetic Vanillin / Vanilla contain Alcohol

Does synthetic Vanillin or synthetic Vanilla contain alcohol? Or is alcohol used in the process of making synthetic vanillin/vanilla? If yes which alcohols are used?---- —Preceding unsigned comment added by 212.22.185.11 (talk) 06:30, 4 June 2010 (UTC)[reply]

I think that vanillin is often dissolved in ethanol - pure vanillin would not contain any alcohol but if you were to buy a bottle of vanilla extract then it will contain 35% alcohol. 86.7.19.159 (talk) 12:21, 4 June 2010 (UTC)[reply]
I have a bottle of synthetic vanilla which contains: water, caramel color (to make it dark like real vanilla), flavors, potassium sorbate. No alcohol. --Chemicalinterest (talk) 12:42, 4 June 2010 (UTC)[reply]
Guys, vanillin is not water-soluble. It has to be dissolved in some sort of (edible) organic solvent or be stabilised with surfactants. John Riemann Soong (talk) 17:27, 4 June 2010 (UTC)[reply]
The "alcohol-free" vanilla flavorings I've seen use glycerol or other "not ethanol but similar-structure" solvent. DMacks (talk) 17:41, 4 June 2010 (UTC)[reply]
Vanillin is water soluble. Not highly soluble, admittedly, but I've regularly made 100mM solutions in water without it crashing out. Rockpocket 18:04, 4 June 2010 (UTC)[reply]
"Solubility in water 1 g/100 ml (25°C)" if we're to believe our own article. DMacks (talk) 18:10, 4 June 2010 (UTC)[reply]

JRS: "Guys, vanillin is not water-soluble. It has to be dissolved in some sort of (edible) organic solvent." Apparently, it's slightly soluble in water.

Page 3-292 of the CRC Handbook contains the following data on vanillin:

No. Name Synonym Mol. Form. CAS RN Mol. Wt. Physical Form mp/°C bp/°C den/g cm−3 nD Solubility
6060 4-Hydroxy-3-methoxybenzaldehyde Vanillin C8H8O3 121-33-5 152.148 tetr (w, lig) 81.5 285 1.056 sl H2O; vs EtOH, eth, ace; s bz, lig

The solubility abbreviations are as follows:

  • i = insoluble
  • sl = slightly soluble
  • s = soluble
  • vs = very soluble
  • msc = miscible
  • dec = decomposes

Other abbreviations:

I have three different bottles of vanilla flavouring in front of me:

Brand Product Ingredients
Dr. Oetker Vanilla – Natural Extract Water, Alcohol, Extract of vanilla
Silver Spoon Vanilla – Natural Extract Water, Ethanol, Extract of vanilla
Silver Spoon Vanilla Flavouring Water, Ethanol, Flavouring

Ben (talk) 14:12, 5 June 2010 (UTC)[reply]

Hey, you must like your vanilla! ;) As for the artificial vanilla mentioned above, potassium sorbate is a mild surfactant, which would help keep vanillin in emulsion (if necessary). Also, depending on the country, it's not always necessary to include ethanol in the ingredients below a certain level, or in certain preparations. In the UK, for example, drinks with less than 1.2% ethanol do not have to be labelled. Physchim62 (talk) 15:43, 8 June 2010 (UTC)[reply]

Mercury and Dental Fillings

Let me preface this question with two things: this is not a request for medical advice, and I do not put any stock in amalgam fillings causing mercury poisoning. However, my question is related to the second point; supposing you took a normal sized amalgam filling and ate it, what would the effects be? What if you inhaled the equivalent ammount of mercury present? Finally, instead of ingesting/inhaling it all at once, what if you stretched out the time span to taking 1/500th of the filling every day, what would be the effects after 500 days (any number can be put in place of 500, I'm really just curious about how dangerous amaglam fillings would be, were the people opposing them correct.) 66.202.66.78 (talk) 10:43, 4 June 2010 (UTC)[reply]

Have you looked at Dental amalgam controversy which while not addressing your hypotheticals would seem the obvious starting point. Nil Einne (talk) 11:12, 4 June 2010 (UTC)[reply]
Mercury poisoning also has a lot of useful information. Between the fact that the mercury is in an amalgam with silver and is thus carried out of the body fairly easily - and the fact that the uptake of mercury through the digestive system is so low (0.01%), this is a really small problem. Mercury is exceedingly dangerous when it's reacted with other stuff - but the pure metal is much less nasty. The amount of mercury you absorb from a tunafish sandwich is likely to be greater than from accidentally swallowing a tooth filling. SteveBaker (talk) 13:06, 4 June 2010 (UTC)[reply]
Mercury is a rather inert metal, so its rate of absorption is low. Mercury compounds are reactive and toxic, and some of them are easily absorbed, making them extremely toxic. Improperly made dental fillings may be at more risk of mercury poisoning. --Chemicalinterest (talk) 14:38, 4 June 2010 (UTC)[reply]
I had a filling like this when I was a kid. I remember nausea and a lot of vomiting. --Ouro (blah blah) 15:56, 4 June 2010 (UTC)\[reply]
That may be better explained by stress or side effects of the anaesthetic, Ouro. The studies I've read show zero history of actual mercury poisoning effects in post-1950s dental fillings. SamuelRiv (talk) 17:33, 4 June 2010 (UTC)[reply]
As I recollect, I got this filling in communist Poland. I surmise that the quality might not have been that high. Or I was young, or stressed. No way to check it out. Just volunteering information, SamuelRiv. But thanks though. --Ouro (blah blah) 19:42, 4 June 2010 (UTC)[reply]
Plus, those really aren't the symptoms you'd expect with mercury poisoning. -RunningOnBrains(talk) 16:31, 6 June 2010 (UTC)[reply]
I swallowed a fresh amalgam filling at the dentists while a teenager. I subsequently got a Master's degree so I hope and assume it had no effect on me. I do not recall felling ill or sick. 92.24.185.225 (talk) 13:26, 6 June 2010 (UTC)[reply]

In case you were interested, no ill effects have ever been shown to be associated with dental amalgam statistically significant; it has been shown, however, that the bis-GMA in composite restorations ("white fillings") is associated with a higher risk of breast cancer in rats. DRosenbach (Talk | Contribs) 23:50, 7 June 2010 (UTC)[reply]

Technological family tree.

Partly a follow on from an earlier disscusion...

Has anyone here come across (in published sources) the idea of a technological family tree?

This being a diagram illustrating which technologies or discoveries are required for a particular technological development to occur.

Technology trees are used in the computer game Civilization and it's sequels and was wondering if these had been used in the academic world as well?

Sfan00 IMG (talk) 13:43, 4 June 2010 (UTC)[reply]

It's not really academic, nor does it involve a formal graphical "tree", but I'm reminded of James Burke's interesting series Connections and its sequels, which traced (some of) the developments that were necessary for particular bits of modern technology to arise. Deor (talk) 14:13, 4 June 2010 (UTC)[reply]
Burke's website here may also be of interest. Ghmyrtle (talk) 15:11, 4 June 2010 (UTC)[reply]
As you can see from his website, though, Burke would be the last person to describe technological advancement in that way as he's spent his professional life illustrating that technology in fact gets developed through all kinds of weird happenstances. He would (and is trying to) make a web, not a tree. Matt Deres (talk) 16:28, 4 June 2010 (UTC)[reply]
Well, the technology trees in Civilization are actually more complicated webs, as I recall. They are greatly simplified from real life, of course. --Tango (talk) 17:03, 4 June 2010 (UTC)[reply]
General histories of technology commonly note breakthrough technology which is the key to some later invention. A powerful compound electromagnet with many layers of insulated wire (Joseph Henry, 1827)) was essential to a practical electric telegraph relay and sounder(one which would function over miles of wire under all conditions), so Morse could not have introduced the practical telegraph before 1827. The phonograph (able to record and play back intelligible speech and music) was invented in 1877, but no breakthrough technology came along just in time as was the case for the Morse telegraph. It could have been built many decades earlier, or as early as good machine tools were available, perhaps in the 1770's, had someone chosen to mechanically impress concentrated sound vibrations on a medium such as wax, tinfoil, lacquer or even soft wood or clay. Edison (talk) 18:30, 4 June 2010 (UTC)[reply]
I think instead of a tree, it would be a directed acyclic graph with all edges pointing in the direction time is flowing. --Rajah (talk) 01:07, 5 June 2010 (UTC)[reply]

RESOLVED! -- Houseplant identification please... -- RESOLVED!

Unfortunately, I haven't a picture. In several stores around China I've seen a small houseplant for sale. It has one very large seed - almost as big as a golf ball - that is dark green and split down the middle from which a stem and a few leaves grow. I can't describe it better than that, but if you've ever seen one that should be enough. I've tried all manner of google searches and haven't hit on a picture. Can anyone out there help me? 91.216.105.31 (talk) 13:50, 4 June 2010 (UTC)[reply]

Are you sure it's a seed, and not some form of bulb? Sfan00 IMG (talk) 13:59, 4 June 2010 (UTC)[reply]
If the seed was red, then it could be a lychee which is a fruit, but I doubt if they are grown as house-plants. CS Miller (talk) 14:15, 4 June 2010 (UTC)[reply]
I've grown avocado seeds before, and the description of the seed fits the bill here. --TammyMoet (talk) 14:48, 4 June 2010 (UTC)[reply]
OP here, on a different IP. I'm sure it's not avocado (I've got one of those growing right now) - this seed is a dark forest green color and smaller than that. It has 2 clear hemispheres. I'm also sure it's not a bulb. If all else fails, I'll try and get a picture on Sunday for you all to look at... no chance before then though. 61.189.63.191 (talk) 15:40, 4 June 2010 (UTC)[reply]
Ok, I went and bought one. http://www.drewsjournal.com/JUNK/mystery-plant.jpg Now someone please help me figure out what it is! My original description stands - golfball-sized seed dark green in color, clear hemispheres. The pot pictured fits in the palm of your hand, so maybe 3" across. Too small and wrong color for an avocado, though it certainly share similarities with that plant. Anyone? 91.216.105.21 (talk) 13:33, 5 June 2010 (UTC)[reply]
It looks to me like some kind of nut, see if this page helps any. Ariel. (talk) 03:32, 6 June 2010 (UTC)[reply]
Yay! One of my coworkers was able to identify it! It's Castanospermum australe, native to the rainforests of eastern Australia! 218.25.32.210 (talk) 05:09, 7 June 2010 (UTC)[reply]

Oil floating under water

News reports say there are "huge plumes" of crude oil from the leak in the Gulf of Mexico which are floating around below the ocean surface. [1] lists densities of various forms of crude ranging from 790 to 973 kg/cubic meter. compared to 1021.98 for "ocean water at 77 F." Tony Hayward of BP claims that"oil has a specific gravity that's about half that of water" per that news story, quite different from the s.g. values cited from the online source. In either case, what force would prevent the less dense crude oil from rising to the surface, or is it uniquely dense, or is the water in the Gulf uniquely less dense? Consider how much force would be exerted by 1 cubic meter of crude at the weakest assumption of 973 density for crude, when it was submerged in seawater at density 1022 kg/cubic meter? Would it be simply the mass difference of one cubic meter (49 kg) times 9.81 m/sec2 or 480.7 Newtons (108 pounds force)? Does the density of the water change more than the density of crude with depth or temperature? Each barrel of oil (.159 cubic meter) should require about 17.1 pounds force to hold it below the surface. If it were in 1 ml droplets or smaller would it somehow not tend to rise? Is it supposed to float under heavier liquid from surface tension/force of habit/? Edison (talk) 16:32, 4 June 2010 (UTC)[reply]

My understanding is that the oil is mixed with gas, which blows it apart into droplets after it escapes. Just as tiny droplets of water may remain suspended in air for a long time, tiny droplets of oil may remain suspended in water for a long time. It's basically what happens when you mix the vinegar and oil to make salad dressing. Looie496 (talk) 16:53, 4 June 2010 (UTC)[reply]
I tried as an experiment releasing 0.1 down to 0.01 ml droplets of motor oil (density about 888 kg/m3 below the surface of a basin of (fresh) water (density about 998 kg/m3. The tiny drops tended to stick to the hypodermic needle, but when I shook it a bit they darted rapidly up to the surface of the water. How small are the "plume" droplets supposed to be, and what principle of physics or chemistry would keep them from rising? So far it sounds like "force of habit." Edison (talk) 16:56, 4 June 2010 (UTC)[reply]
I think it's a result of the dispersant they've been adding to the oil. It causes the oil to form droplets rather than a layer, and those droplets end up suspended in the water. I don't know the details. --Tango (talk) 17:01, 4 June 2010 (UTC)[reply]
If the droplets are down in the "few micron diameter" size then maybe they are in fact rising but some frictional effect of moving the water apart impedes the rise to a slow rate and some surface tension effect impedes their coalescing into larger drops with a greater terminal velocity of rise. Edison (talk) 17:06, 4 June 2010 (UTC)[reply]
Yes - upward force is proportional to cube, 'drag' proportional to square - so smaller drops should accelerate faster slower - though I expect there's a peak velocity in water as there is in air. Also viscosity
Don't think any surface tension effect would prevent coalescing though - unless when as a micelle
Perhaps there are down currents in that area that keep the drops underwater for longer ???87.102.32.39 (talk) 19:18, 4 June 2010 (UTC)[reply]
Keep in mind too that the crude is coming out pretty hot so it may also be partly fractionating, so the light ends will separate and head more quickly to the surface. The s.g. of vacuum tower bottoms is 0.97 to 1.03.[2] Admittedly, that is the very heaviest fraction. Franamax (talk) 17:19, 4 June 2010 (UTC)[reply]
If it so hot that it is "self refining" in a fractionating or cracking process, then why is it simultaneously so cold that it freezes up and clogged the removal dome? And the sludge left in a refinery (if that's what "vacuum tower bottoms " is) is still far lighter than seawater. For the stuff to stay in an emulsified blob below the surface, the blob would have to have a higher density than the water, or the entire column of water would have to be part of the emulsion. Edison (talk) 17:29, 4 June 2010 (UTC)[reply]
First of all, the crude is what's hot, it's the ocean water that is very cold, it is mixing with the gas portion of the outflow to form gas hydrates, that is what is clogging the containment domes. Second, the MSDS I linked for bottoms (it's only sludge when it's really hot, it's asphalt when it cools) says the specific gravity is 0.97-1.03, this [3] shows seawater wirh a s.g. of 1.025, so are you saying 1.025 >> 1.03? Or have you done all the lookups for temperature and pressure and you have a derivation for "far lighter than"? Lastly, all crude oil is "self-refining", that's why it stinks. The lighter fractions are separating preferentially and the benzene is making you dizzy. Crude oil is not one "thing", it is a mixture of many different compounds, each with a different partial pressure. There is certainly no cracking process at work, but the light ends will definitely get to the surface (and evaporate) faster than the heavies. That's why they've been discussing the problem with burning-off once the slick has been floating around more than a few days. Franamax (talk) 03:56, 5 June 2010 (UTC)[reply]
Press reports, such as one from the New York Times say one plume is "3 miles by 10 miles by 300 feet," which would be 7.5 x 10 11 cubic feet, which would be 133 x 109 barrels of oil. The max leak rate estimate is 80,000 barrels a day, or about 3.5 million barrels total so far. Thus there appears to be a gross overstatement of the actual mass of oil involved in the plumes. A scientist states it might have the "consistency of thin salad dressing" in places, fostering the impression that far more mass of oil is in the plumes than seems likely. Edison (talk) 17:44, 4 June 2010 (UTC)[reply]
The media can also be quick to jump on a sensational story and slow to admit error. The laws of physics are hinting to me that this is what is going on. Googlemeister (talk) 18:26, 4 June 2010 (UTC)[reply]
Two things - when the Times gives the size of a plume, they're likely to be describing the volume in which the concentration of oil is appreciably higher than zero — not the volume of pure oil. Compare with coverage of the 2010 eruptions of Eyjafjallajökull, where our article discusses an ash plume more than 8 km high. The ash is there, it's visible, it's definitely occupying that volume — but no one assumes that there's a solid inverted cone of ash more massive than the volcano beneath; there's air mixed in. Indeed, the vast majority of the ash plume is air, just as the vast bulk of these oil plumes is seawater.
A second point that the ash also illustrates nicely is how long it takes for fine particles to settle. Air and volcanic ash particles have a much greater difference in density than oil and water, and air is a far, far less viscous medium than water — but volcanic ash still managed to stay suspended in the atmosphere for days and travel for hundreds of miles. I applaud your experimental spirit in your motor oil and water test, but if the oil is being forced out of the borehole and is undergoing turbulent mixing, then the droplets produced – at least some fraction of them – are apt to be significantly smaller than the 10-100 microliter volumes you tested. Using the creeping flow approximation to calculate terminal velocity, and assuming a density difference of 200 kg/m3 between oil and water one can estimate the rate at which oil droplets will rise to the surface. A sphere 1 mm in diameter (which is quite distinctly visible, though it only has a volume of about 0.5 microliters) would have a terminal velocity of just 3 millimeters per minute; the effect of its bouyancy would in most cases be dwarfed by the effect of rising or falling ocean currents. At that speed, droplets deposited at the borehole depth of 1500 meters would take about a year to reach the surface. TenOfAllTrades(talk) 20:11, 4 June 2010 (UTC)[reply]
One of the biggest sources of error here is accurately estimating the density of the oil that is coming from the reservoir. It is at least plausible that some of the oil in the Gulf of Mexico is on the order of 10° API - in other words, heavy oil - in which case it is not less dense than water. Deep below the sea surface, there is not much turbulence, but as the plume jets its way out of the bore-hole (or some other place where it is leaking), it is probably spraying out in a very turbulent way. This may complicate its path to the surface. If the pressure were low, the plume would rise as a laminar flow to the sea surface (where wave action would turbulently mix it in the top few feet of water). But since the borehole pressure was very high (enough to cause a blowout), it is hard to predict exactly what happens as the oil contacts the open ocean at the sea bottom (or other leak location). As has been pointed out above, the situation is further complicated because the fluid that is leaking from the reservoir is a mixture of crude oil, natural gas, sediments, brine, drilling mud, drilling crud, and other impurities. Nimur (talk) 19:17, 4 June 2010 (UTC)[reply]
Heavy crude oil describes that product as bitumen or oil sand. Is there any evidence that was in the deposit they drilled into, or that it is found in that area of the Gulf? It sounds like heavy crude does not flow well in general. Edison (talk) 20:20, 4 June 2010 (UTC)[reply]
[4] "What we're doing is we're taking a highly pressurized solution, and we're exploding it out of the sea floor under high pressure and high velocity."
expand assuming the oil is tar+oil+gas then at lower pressures the gas will boil off .. pressure increases with depth - it's possible therefor - that as the mixture rises further de-gassing (of methane/ethane etc) happens. It's possible again that this degassing is sufficiently violent (especially if it becomes supersaturate with gas before boiling) that this loss of gas causes the droplets to become yet smaller, again reducing the speed at which they rise. It's also possible that this doesn't happen or the effect is negligable.
I didn't actually see anywhere a claim that the liquid is heavier than water though. Density of 0.8 - 0.9 cc takes quite a long time to rise hundreds of feet - and there may be strong lateral currents.
Did you misread the CEO ? :

Mr. TONY HAYWARD (Chief Executive Officer, BP): There's no evidence of (that) .. (the) oil has a specific gravity that's about half that of water. It's very difficult for oil to stay in the column. It wants to go to the surface because of the difference in specific gravity.[5] [6]

As regards the density vs. depth question compressibility is the thing you want this compares toleune and water (tolune is ~twice as compressibile at 8.94 x 10-4 MPa-1) - so maybe there is something in this idea.. but it's a very small figure (no idea what the pressure is down there) There should be better tables somewhere on the internet.87.102.32.39 (talk) 19:56, 4 June 2010 (UTC)[reply]

87.102.32.39 (talk) 19:27, 4 June 2010 (UTC)[reply]

Hayward seemed to be trying to say that the oil has a low specific gravity and that there was no evidence of plumes (despite the fact that he was speaking in a nonsentence which could be taken to mean the opposite). Edison (talk) 20:22, 4 June 2010 (UTC)[reply]
Uh - yes - I think they mistyped in the link is used (corrected it) - looks like they've joined two sentences together - but it does look like he was saying oil has half the density of water. (The english have acquired the bad habit of lying/stretching the truth in the last decade - I know . live there.)..87.102.32.39 (talk) 20:48, 4 June 2010 (UTC)[reply]
Several news sources ([7] [8] [9]) have used the terms "heavy crude" and "tarballs". It is not clear whether this terminology is scientifically accurate or media hype. Heavy oil is definitely known to exist in the gulf, and indeed is the primary constituent of the geologically similar Orinoco Belt. Biological activity in the deep subsurface and the origin of heavy oil (Nature, 2003), states: "Biodegraded oils also represent a significant fraction of the petroleum in conventional oil reserves and will be common among future oil discoveries likely to be made in deep-water areas of the world (for example, the Atlantic margin basins of Africa, South America, Canada and the Gulf of Mexico)." Reservoir geochemistry of South Pass 61 Field, Gulf of Mexico: compositional heterogeneities reflecting filling history and biodegradation (Organic Geochemistry, 1996) has tables of geochemical analysis. A quick look through some of my Oil & Gas Journal archive has a bunch of market report stories on "heavy" crude in Gulf of Mexico - I'm looking at a February 2010 issue talking about 14 and 17° oil production in Louisiana. Those would still be lighter than water, though. Unfortunately, the exact chemical makeup of the Macondo is pretty intensely "company confidential" and I'm sure only privileged members of BP's production team know what's coming out of the reservoir. Nimur (talk) 20:56, 4 June 2010 (UTC)[reply]
John Ebdon, an Emeritus Professor and chemist at Sheffield University said in a letter in Friday's Guardian (can't find a link): "...True, seawater has a specific gravity of about 1.03g/cubic cm whereas that of crude oil is about 0.8g/cubic cm ... but significant fractions of crude oil consist of asphaltenes and other complex substances with specific gravities around 1.4g/cubic cm ... these heavier fractions will almost certainly separate and so be retained at depth." --Tagishsimon (talk) 04:11, 5 June 2010 (UTC)[reply]
The improvised refinery which fractionates the oil to create the asphalt/bitmen submarine is of dubious plausibility. Asphalt of dubious 1.4 density "retained at depth" would not wash up on Pensacola Beach and would not be much more of an issue than if it had not been brought up by drilling. The media creates spectres of "plumes" of oil many orders of magnitude greater in mass than could possibly exist. Edison (talk) 04:41, 5 June 2010 (UTC)[reply]
Scientists now say that microscopic drops of oil are a natural part of ocean water, and that bacteria digest it and eventually are feed for fish. Edison (talk) 01:51, 8 June 2010 (UTC)[reply]

Doorbell Continually Hums or Buzzes

the button is stuck in the contact position. how long till the electromagnet will burn out and the bell unit won't work —Preceding unsigned comment added by Alexsmith44 (talkcontribs) 17:30, 4 June 2010 (UTC)[reply]

Why not just disconnect and repair it, rather than go through the discomfort of waiting for it to burn out. [10] which I suspect you've already read as it's linked to from a page almost identical to your question, gives you a description of how to dissemble a doorbell. Regards, --—Cyclonenim | Chat  17:37, 4 June 2010 (UTC)[reply]


can you just tell me how long till it burns out? i dont have the tools right now. —Preceding unsigned comment added by Alexsmith44 (talkcontribs) 17:50, 4 June 2010 (UTC)[reply]

It depends, but "long" is a good estimate. Think weeks or months, or even years. There is no particular reason why anything should "burn out". --Stephan Schulz (talk) 17:57, 4 June 2010 (UTC)[reply]
A defective doorbell circuit is likely to cause radio, television and celphone interference, which is one good reason to get it fixed. Some doorbell transformers have a thermal protection element which heats up and opens the current , then resloses, providen a spark each time which interferes with electronic communication. The thermal protector often burns out and just arcs continuously, provide ongoing interference. In the US, it is a violation of the FCC rules to create "harmful interference". A shorted transformer could also start a fire, [11]. (Paradoxically, in a great many instances over the years, doorbell wires shorted by a house fire have alerted the family and saved their lives).How long would it take to burn out? I once read an account of a person growing up in an apartment with a defective doorbell circuit. The transformer or chime hummed all the time, 24/7, except when someone pressed the doorbell, at which time it quit humming momentarily, and they knew someone was at the door. Edison (talk) 18:00, 4 June 2010 (UTC)[reply]
Doorbells are often very high-current devices - you may be burning two or five amps if the electromagnet is constantly activated. (Some are lower-power - here's a 30 watt bell). Most bells are not intended to be "energy-efficient", because you want the bell to be loud and momentary - so effectively, they are designed to consume on the order of 50 or 100 watts, (but quickly shut off). If the switch is stuck in contact-position, and you are burning a steady 50 or 100 watts, over a month that will add up) and will cost you in the neighborhood of $5, depending on your electricity price. Nimur (talk) 19:08, 4 June 2010 (UTC)[reply]
That makes me wonder, is there a danger that it was installed with wiring that can't handle constant use? APL (talk) 20:45, 4 June 2010 (UTC)[reply]
Actually, since you mention it - that's certainly possible in older houses. In the UK, there is stuff called "bell wire" that used to be used specifially for wiring up doorbells and those bell systems in big houses that let the lady or gentleman of the house summon servants from the kitchen from any room in the house. That wire was very thin indeed, single-core with enamel paint insulation. Bell wire was the home-experimenters wire-of-choice back when I was a kid because it was cheap and you could get it in any electrical repair store. So it wouldn't surprise me at all if it overheated and burned out in an old property...and that's a fire risk. But I'd expect chunkier wiring in modern doorbells - so it's probably not a problem for a house that's less than maybe 50 or 60 years old. SteveBaker (talk) 04:25, 5 June 2010 (UTC)[reply]

can any of these radio frequencies harm me?

No. APL (talk) 20:45, 4 June 2010 (UTC)[reply]
Really - you can trivially repair this with no more tools than a screwdriver - or (if desperate) a kitchen knife - or maybe even a thin coin! Waiting - on the offchance that it'll just blow up and "fix itself" - is pretty pathetic. Turn off the power, unscrew the bell button or pop open the cover, if the contacts inside look corroded then just scrape off the surface corrosion with your knife/screwdriver - then bend them so they don't quite touch when the button isn't pressed, and do touch when it is pressed. Reassemble the button and turn the power back on - and you're done. It couldn't possibly take 10 minutes to fix. SteveBaker (talk) 23:12, 4 June 2010 (UTC)[reply]
Curiously enough, there was "bell wire" before electricity was in use. Great houses in the 18th century, including the White House in John Adams' Presidency had "bell pulls" to signal servants that someone in a certain room wanted a servant to come. The "bell wire" was uninsulated copper, brass, or iron running to the kitchen or "Servant Central" where a pull on it rang a bell. Edison (talk) 04:31, 5 June 2010 (UTC)[reply]
Yes - I noticed that when I searched Wikipedia for bell wire and found it mentioned in Bell pull. It did surprise me to see the term used that way. I wonder whether these old houses went from physically pulling the wires to passing electicity down them when they converted to new-fangled electrical systems (maybe using an earth as the return path). That would save ripping out the walls to put in the new wires. But the stuff I remember having as a kid in the late 1950's was single-core copper, very brittle and coated with what looks like a brown enamel paint or maybe a dark varnish...that was called "bell wire" too. The electrical projects in "Boy's Own" style books always specified either "Bell wire" for battery-powered things or "Lamp wire" where higher current was required. This definition on Answers.com confirms my memory. SteveBaker (talk) 04:56, 5 June 2010 (UTC)[reply]

The difference between fire and plasma

Take this image for example, it's a projectile fired from a railgun. But of course, there are no explosives involved so that trail behind the projectile is plasma and not fire correct? So is it fair to say that plasma and fire can look the same sometimes, but are two different things? ScienceApe (talk) 18:06, 4 June 2010 (UTC)[reply]

You don't need explosives for flame; just heat, oxygen, and something to burn. --Sean 18:53, 4 June 2010 (UTC)[reply]
(ec) Most of the time, ("day-to-day" interpretation), when you think of fire, you are thinking about two things: (1) an oxidation reaction between oxygen in the air and carbon (or something else) in the fuel; and (2) a gaseous semi-ionized plume of hot air that is incandescent - glowing reddish yellow because it is so hot. The chemical reaction (1) provides the energy and heat, and contributes some chemical constituents to the flame plume, but mostly the glowy part is just the ambient air getting heated up (and indeed, somewhat ionized). It would be fair to call that flame plume a "plasma" - but more specifically, it would be a dense weakly-ionized plasma. In other words, it is more "gas-like" than "plasma-like", in that thermal interactions dominate the behavior, rather than electromagnetic interactions. Typically, in plasma physics, we like to think about "sparse" (i.e., nearly a vacuum) plasmas where the collisions between molecules and ions are very rare. But plasmas can be created in many ways - anything which provides enough energy to ionize a gas sufficiently that its electromagnetic effects are non-negligible. That can be a chemical flame, or any other energy source.
All this being said, let's analyze the railgun photo you linked. Unfortunately, the caption/description is not really very explanatory - it says the gun is "electromagnetic," but that doesn't necessarily mean that there was no explosive (a lot of railguns use a primer charge to create a super-compressed gas in preparation for additional energy). It's also possible that at 2000 m/s, the hypersonic projectile is tearing up its surrounding air so much that the thermal energy imparted to the shock cone is enough to incandesce or even ionize. To what extent is the shock cone ionizing? To what extent does the electromagnetic propulsion system ionize the surrounding air? To what extent does the primer-charge interact with the muzzle blast? How do these parameters affect the muzzle ballistics, kinetics, trajectory, and the energy balance for the projectile? All of these things would be interesting research questions, and I would bet that the NSWC guys are instrumenting the heck out of this apparatus to answer exactly those kinds of questions. But we only have the image caption - without further information, it's not easy to answer definitively. But keep in mind - a plasma is just a gas that has been ionized so much that the electromagnetic effects are dominant, whether it's ionized by energy from a chemical fire, thermal collisions, electromagnetic radiation, nuclear energy, hypersonic shock, or other energy source. Nimur (talk) 18:57, 4 June 2010 (UTC)[reply]
chemisty I've always understood most flame/fire to be primarily a radical reaction - also high energy but not a "full" plasma since many of the entities are neutral. If the flame is hot enough though gas will be ionised.. answer? it's a borderline case depending on T and gases burnt as well.87.102.32.39 (talk) 19:38, 4 June 2010 (UTC)[reply]
(EC)Plasma and a fire can look alike. Both can involve very hot gases. See Plasma (physics), which says that a plasma is electrically conductive because of ionization. But flame and smoke can also be conductive, depending on what the chemical composition is. Edison (talk) 19:01, 4 June 2010 (UTC)[reply]
If you look at the video that this shot was taken from, I think that it appears to be flame, not plasma. J.delanoygabsadds 19:05, 4 June 2010 (UTC)[reply]
I can't tell - but yellow air plasma isn't usual. A plasma should decay without any soot etc. Maybe there was some coating on the 'bullet' that caused the plasma to go yellow?87.102.32.39 (talk) 19:45, 4 June 2010 (UTC)[reply]
You're right, for it to glow that brightly there must be something more than just incandescent air. With a flame, there are tiny particles of soot glowing (if you have complete combustion with no soot then you don't get a yellow flame, you get a dim blue flame that isn't caused by incandescence). There must be something similar in that plume. --Tango (talk) 19:54, 4 June 2010 (UTC)[reply]
This sounds 'clu-ey' [12] "Particle debris ignites as a test slug exits the Office of Naval Research (ONR) 32 MJ (megajoules) Electromagnetic Railgun (EMRG) laboratory launcher located at the Naval Surface Warfare Center Dahlgren Division (NSWCDD)." 87.102.32.39 (talk) 20:04, 4 June 2010 (UTC)[reply]
Also, we have no way of knowing whether the photo is a true-color or false color image. I'm leaning towards a Schlieren photography, Rapatronic camera, polarizing filters, or some other "weird" photographic trickery (not even counting post-production digital color rescaling!) Note that the timestamp has microsecond precision. Such high-speed cameras often are monochromatic. Note how you can see the shock cone - that's a dead giveaway for a shadowgraph or Schlieren system. So, we can't say for sure whether the yellow and green in this photograph are "meaningful" for analysis. Nimur (talk) 21:11, 4 June 2010 (UTC)[reply]
There's an even more clear image of the shock cone at this link (scroll down) [13] 87.102.32.39 (talk) 21:30, 4 June 2010 (UTC)[reply]
Horse's mouth [14] "The flames are from pieces of the projectile disintegrating; the 7-pound slug is jammed so firmly between the rails that when it’s fired, pieces shear off and ignite in the air."
one commenter says "...but the video is slightly out of order... the first shot is of the impact. If you pause the video around 6-7 seconds you can see the projectile only has a plasma trail until it strikes the target." (same link) this isn't quite right the image above is earlier in time than that in which the projectile flys between gun and target.
The linked video has time count in the lower right . so if doubtful you can check for yourself. 87.102.32.39 (talk) 21:49, 4 June 2010 (UTC)[reply]
This image is confusing File:EMRG 070128-N-0000X-001.jpg - apparently showing a very similar moment in time - but a different colour - since the slug is aluminium and the fire is supposed to be from particles from the slug I would assume that the photo is a better colour representation. So why is the video footage (and the frame above) yellow?87.102.32.39 (talk) 22:12, 4 June 2010 (UTC)[reply]

Archimedes principle???

if a body is immersed in a fluid,is the volume of the displaced fluid equal to the volume of the body???......... —Preceding unsigned comment added by Researcher02 (talkcontribs) 19:38, 4 June 2010 (UTC)[reply]

Only if it sinks - this can be demonstrated with a full bathtub, large amounts of polystyrene foam, and some lead shot..
The density of the body must be more than the density of the fluid.87.102.32.39 (talk) 19:42, 4 June 2010 (UTC)[reply]
(e/c) under conventional assumptions (non-viscous fluid, unconstrained and not otherwise subject to compression, non-bouyant, non-dissolvable object, etc), yes. The fluid will conform to the shape of the object (so displacement will not be greater than the volume of the object). note that for hollow objects, displacement will differ according to whether the interior spaces are open or sealed - open spaces will fill, making the displacement equal to the volume of the solid material; sealed spaces will displace both the volume of the solid mass and the volume of the sealed space. --Ludwigs2 19:48, 4 June 2010 (UTC)[reply]
(ec) The volume of the displaced fluid is equal to the volume of that part of the body that is in the fluid. If the object sinks, that will be the whole body. If the object is floating then it will only be part of the body (a part with volume equal to the volume of the fluid that has the same mass as the object). --Tango (talk) 19:49, 4 June 2010 (UTC)[reply]
That assumes that the fluid displaces all the air in the body - if air is trapped (closed-top, open-bottom area or by hairs, etc), then the trapped air has to be counted as part of the body (after compression). CS Miller (talk) 13:33, 5 June 2010 (UTC)[reply]
The article section about this is Buoyancy#Archimedes' principle. Comet Tuttle (talk) 21:09, 4 June 2010 (UTC)[reply]
The OP's question does not concern Archimedes' principle. The OP did not ask about a floating body. Cuddlyable3 (talk) 22:41, 4 June 2010 (UTC)[reply]

Why did my bonfire burn 'blue'?

Last summer, I used a bonfire to dispose of a large quantity of waste wood (mainly laurel branches) in my garden..

I noted that, when started the fire was rather yellow and produced a lot of smoke, but as it got going it smoked a lot less and changed from a yellow flame to a more blueish and less visible one...

What might be the cause of this?

In addition, a previous bonfire in a previous year apparently managed to make a small quantity of charcoal, although I'm not entirely sure how... The same bonfire for whatever reason was also able to be 'relit' merely by placing suitably dry fuel on the ashes the following morning, which came as a quite a suprise...

Anyone care to suggest a possible explanation for the charcoal production? or for the 're-light' capability? 62.56.112.251 (talk) 20:19, 4 June 2010 (UTC)[reply]

It's very common for a wood fire to create coals that stay burning hot overnight, in some cases they can continue to burn for days. That article describes them better, but briefly: after the outside of the wood burns, and the more volatile componants of the wood have escaped and burned, the heavier carbon based solids in the innermost parts of the wood stick around because they haven't been properly exposed to the air, but at the same time they're not volatile to evaporate through the hot wood. Usually these componants burn very slowly making hot coals. (These are good for cooking over, because of their even heat.) If they go out before they're entirely spent I suppose you'd have charcoal. (Which does not come from true coal. A lot of people are confused on this point.)
Hot coals can stay hot for a suprisingly long time because they're covered in a layer of light, fluffy ash that both insulates them and prevents very much oxygen from reaching them. That way they don't cool off, but they also don't burn up very quickly. APL (talk) 20:39, 4 June 2010 (UTC)[reply]
Indeed - a fairly large bonfire will usually have hot embers left for several days. --Tango (talk) 21:12, 4 June 2010 (UTC)[reply]
Follow on question, Would this 'coals' method be exploitable for low-level metalworking? Sfan00 IMG (talk) 21:55, 4 June 2010 (UTC)[reply]
Did you mean could the charcoal you made be used for metalworking? I see no reason why not - it was used in the past,as the article charcoal says. 87.102.32.39 (talk) 03:21, 5 June 2010 (UTC)[reply]

Hmm. Have you seen She? --Trovatore (talk) 20:42, 4 June 2010 (UTC)[reply]

I would think that at first the oxygen supply was limited. After a while it started burning more completely, producing the hotter blue flame and less smoke. --Chemicalinterest (talk) 20:54, 4 June 2010 (UTC)[reply]
The yellow flames are incomplete combustion. The blue flames are complete combustion. If everything burns cleanly then there are no soot particles to glow red-hot, which is what gives the flame its colour. --Tango (talk) 21:12, 4 June 2010 (UTC)[reply]


June 5

What is the reduction potential for this reaction: 2 NO3- + 12 H+ + 10 e- ←→ N2 + 6 H2O Thank you. --Chemicalinterest (talk) 00:28, 5 June 2010 (UTC)[reply]

from [15]
NO3- > NO 0.957
NO > N2O 1.591
N2O > N2 1.766
I think you can just add those ~2.2V ? Maybe not - different number of N atoms... Different number of electrons transfered in each reaction.
It's {(0.957x3)+(1.591)+(1.766)}/5 (reason for that formula at Standard_electrode_potential 87.102.32.39 (talk) 01:20, 5 June 2010 (UTC)[reply]
About 1.25V also see here [16] 87.102.32.39 (talk) 01:39, 5 June 2010 (UTC)[reply]
Thanks for your help. (BTW, when you type chemical formulas, it is recommended to put subscripts and superscripts around the appropriate figures, such as NO<sub>3</sub></sup>-</sup>, making NO3-) I don't think it is 5V. Even F2 gas is +2.87 or +3.01. Many times going from step to step is harder than "taking all the steps". Hypochlorite, chlorite, chlorate, perchlorate is an example. Each one has an oxidation potential of about +1.5V to the next one, yet perchlorates are not supremely reactive because the steps are not added up (they are actually the least reactive). +1.25V seems right, a little higher than the reduction potential (+1.23) for oxygen in an acidic environment. --Chemicalinterest (talk) 11:07, 5 June 2010 (UTC)[reply]
Yes the EMF is an average - that comes from the energies of reaction being additive, but the actual EMF's are measured 'per electron transfered' - thus energy change = electrons x voltage
eg for NO3- >> NO (change from V state to II state)
eg NO3- + 4H+ + 3e >> NO + 2H2O + NO (3 electrons)
..the energy change is three times that what it would be if only one electron had been transfered
this equation in fact :ΔGcell = −nFEcell where F is Faraday constant
It's actually similar at a simple level to the formala for potential energy of a charged particle in an electrostatic potential E=QV (or as written in Electrostatic_potential#In_electrostatics UE = qV ) 87.102.43.94 (talk) 11:41, 5 June 2010 (UTC)[reply]

Who first hypothesized/discovered/elucidated gluconeogenesis?

Who first hypothesized or discovered or elucidated gluconeogenesis? Thank you. --Rajah (talk) 00:54, 5 June 2010 (UTC)[reply]

this link mentions some key people (last 2 paragraphs) 87.102.32.39 (talk) 03:17, 5 June 2010 (UTC)[reply]
Thanks, but that's about glyceroneogenesis, which is a subset of gluconeogenesis. The conversion of pyruvate to glucose would be the major component of gluconeogenesis. --Rajah (talk) 06:52, 5 June 2010 (UTC)[reply]
Sorry, I was working quite late - didn't even notice at the time. Please accept my apologies.87.102.32.39 (talk) 10:51, 5 June 2010 (UTC)[reply]
No problem. Actually, your mention of it spurred the creation of the article on glyceroneogenesis! --Rajah (talk) 17:02, 7 June 2010 (UTC)[reply]
This is not a topic I know anything about, but just for the hell of it I trotted out my Googling skills, and found that the fundamental work on the chemistry was done by A. I. Ringer and colleagues in a series of papers in the Journal of Biological Chemistry in 1913 and the following few years. The physiological background was worked out over the previous few decades, largely in German publications. The state of affairs as of 1913 is reviewed in this paper. Looie496 (talk) 22:33, 5 June 2010 (UTC)[reply]
Awesome, thanks! Yeah, tha'ts probably the first mention in English. And a European would've probably been the first to hypothesize it under a different name. --Rajah (talk) 17:02, 7 June 2010 (UTC)[reply]

Brass

Alloys of Copper and Zinc create a how to hop metal brass.lam not work on this solution and HCl solution H2SO4 ? —Preceding unsigned comment added by I love chemistry (talkcontribs) 04:02, 5 June 2010 (UTC)[reply]

Could you please rephrase your question, I don't really get what you're trying to do. 67.170.215.166 (talk) 04:30, 5 June 2010 (UTC)[reply]
Sounds like "I love chemistry" is asking how to separate Cu and Zn in brass using HCl and H2SO4, which sounds a little like a homework problem, though I won't assume, but I recommend checking reduction potentials and noting if any salts will precipitate. SamuelRiv (talk) 05:41, 5 June 2010 (UTC)[reply]
This is original research. I tried dissolving brass in hydrochloric acid but it is very difficult too, it is a rather noble alloy. Do not expect the zinc to come fizzling out, leaving a framework of copper behind. --Chemicalinterest (talk) 11:08, 5 June 2010 (UTC)[reply]
See galvanic series; you can see that it is rather noble (closer to the top than say, aluminum, iron)--Chemicalinterest (talk) 11:15, 5 June 2010 (UTC)[reply]
More so than either copper or zinc - I don't really understand why.87.102.43.94 (talk) 11:27, 5 June 2010 (UTC)[reply]
If the aim is an analysis of Cu and Zn in brass then this link gives the standard proceedure [17] or this 87.102.43.94 (talk) 11:25, 5 June 2010 (UTC)[reply]
If you want to know how to make brass there is an interesting but old manual here [18] eg "Put 4 1/2 lbs. of copper into a crucible, expose it to heat in a furnace, and when perfectly fused add 1 1/2 lbs. of zinc. The metals will combine, forming that generally used alloy called brass." there are recipes for other brasses too. And a more up to date list at Brass#Brass_types ? 87.102.43.94 (talk) 12:09, 5 June 2010 (UTC)[reply]

Asa Gray and John Edward Gray

I'm planning on editing a few articles on nineteenth century biology and I've come across a question I can't answer. Is there a relationship between Asa Gray and John Edward Gray? I know one is British and the other American, but they both seem to be biologists during the same time period. I'm most interested in if they are related biologically and if they there are any primary sources mentioning the relatedness of both of them. Thanks a bunches :) Peter Napkin Dance Party (talk) 07:27, 5 June 2010 (UTC)[reply]

The WP:RD/H may be a better place. --Chemicalinterest (talk) 11:09, 5 June 2010 (UTC)[reply]
Here'll do. It's not looking very likely, or at least not in any meaningful span of generations. AG is described as being "of Scotch-Irish descent" [19] whilst JEG's grandfather was a London seedsman who died without inheritance [20]. I'd question how much migration of "Scotch-Irish" into the London area was going on in the mid or early C18 ... not very much. All in all, has the feel of a coincidence of surnames and nothing more. --Tagishsimon (talk) 13:02, 5 June 2010 (UTC)[reply]

Brass (30% Zinc and 70% Copper)

How to Brass (alloy of 30% Zinc and Copper 70%) not destroyed in the dense environment axid Chlohydric, axid in solution and concentrated sulfuric (H2SO4)and water Amonia (NH3). No surface coating of the alloy, only add other elements to increase resistance axid, but no change yellow color of the alloy elements nay.Neu you know is right, then please just help with!--I love chemistry (talk) 13:40, 5 June 2010 (UTC)[reply]

Your question does not seem very clear. Do you want to know why acids don't attack brass? --Chemicalinterest (talk) 13:41, 5 June 2010 (UTC)[reply]
If that is your question, then my explanation is that: On the surface, there is a mixture of copper and zinc. The small amount of zinc corrodes, leaving only copper on the surface (explains why few bubbles appear on surface when immersed in acid). The copper is resistant to acid attack (see standard reduction potential for explanation of oxidation and reduction, also see redox), which prevents further corrosion. Brass is also resistant to corrosion by the oxidizing properties of copper salts. Again, the small amount of zinc on the surface dissolves, plating the brass with a durable coating of copper metal.
In short, when the zinc dissolves, the brass is essentially a copper-coated strip of brass. --Chemicalinterest (talk) 13:48, 5 June 2010 (UTC)[reply]
Actually in ammonia the copper actually dissolves. In air, copper forms a protective coating of copper(II) oxide. That dissolves in ammonia to form a complex similar to tetramminecopper(II) sulfate. A new coating forms and dissolves, eating away the copper. This happens very slowly though. It also happens in acid, albeit very very slowly. --Chemicalinterest (talk) 14:10, 5 June 2010 (UTC)[reply]
If you wanna dissolve brass, you could stick it in nitric acid, and that will dissolve it easily. (Getting the nitric acid might be hard, though -- it's a restricted chemical because it can be used to make bombs.) FWiW 67.170.215.166 (talk) 01:42, 6 June 2010 (UTC)[reply]
Simpler way; mix 3% (household) hydrogen peroxide and household muriatic acid in about a 1:1 ratio. That should dissolve brass, the H2O2 functioning as an oxidizer and the HCl dissolving the oxide produced so the peroxide can continue oxidizing the metal. --Chemicalinterest (talk) 12:05, 6 June 2010 (UTC)[reply]
It does not react that way. Acids stabilize H2O2. --Chemicalinterest (talk) 00:32, 10 June 2010 (UTC)[reply]
This might also be a good way to make a binary chemical weapon: H2O2 + 2HCl → 2H2O + Cl2. Don't try this at home (at any rate, not unless you do this in a fume hood and while wearing a gas mask). And if you do get the results that I've predicted, Mr./Ms. I-Love-Chemistry, don't blame me. 67.170.215.166 (talk) 00:51, 7 June 2010 (UTC)[reply]
Chemicalinterest, didn't your chemistry teacher tell you not to mix halides with strong oxidizers? 67.170.215.166 (talk) 01:09, 7 June 2010 (UTC)[reply]

The simple answer to your question is that an alloy doesn't react in the same way as the metals that make it up. You can see this very well with stainless steel, which is mostly iron but is much more resistant to oxidation. There is a huge amount of research as to which alloys have different properties and why, and I can't summarize it all here; there are different answers for different alloys, and it's a specialized area of research. Physchim62 (talk) 14:56, 7 June 2010 (UTC)[reply]
I leave it as an exercize for the readers why the "binary chemical weapon" wouldn't work. Physchim62 (talk) 14:56, 7 June 2010 (UTC)[reply]

Acid stabilizes hydrogen peroxide so it does not decompose. Also the Cl- ion is resistant to oxidation by the O22- ion. --Chemicalinterest (talk) 00:33, 10 June 2010 (UTC)[reply]

Information on Planting lilies in green house

Dear sir I am asking for information about planting lilies in green house

  1. first steps in soil ( compost & per light )
  2. soil ( EC & PH )
  3. fertilizer program ( 2 weeks 2 – 1 – 1 & 6 weeks 1 – 1 – 1 )

Spatially fertilizer in ( first – medal – end ) life
I am from medal est. ( UAE ) & I will planting on 1st October <E-mail address removed> thank you
hamid —Preceding unsigned comment added by 94.59.56.184 (talk) 13:52, 5 June 2010 (UTC)[reply]

Hi Hamid, we don't respond to queries by email so I've removed your address. Brammers (talk/c) 18:39, 5 June 2010 (UTC)[reply]

You question does not say which type of lily you are going to grow. You can see from the article that there are many species and some require special conditions to succeed. Some lilies require deeper planting because they develop roots from the stems, others need a deep soil with a 'woodland' shade. Of course many will grow in a nice soil with compost, some slow-release fertilizer and regular watering. My experience with lilies is that you don't need a complicated watering/fertilizing programme. If you are going to grow them commercially then you will need specialised information. Richard Avery (talk) 07:32, 6 June 2010 (UTC)[reply]

Deepwater Horizon valves

Hi. Could a piston valve assist in one of the procedues used to cap the flow of oil or to redirect it into a ship? Thanks. ~AH1(TCU) 14:38, 5 June 2010 (UTC)[reply]

A Piston valve could indeed help - in fact the automatic shutoff valves that failed (the Blowout preventers) causing the problem are (or can be) a type (or variation of) of piston valve (in that the closing motion is linear).87.102.43.94 (talk) 15:05, 5 June 2010 (UTC)[reply]
A Gate valve has some similarities in design and would be another good choice for closing of a wide pipe.87.102.43.94 (talk) 15:13, 5 June 2010 (UTC)[reply]
Has anybody suggested this idea to BP? ~AH1(TCU) 15:22, 5 June 2010 (UTC)[reply]
They've been cutting through the pipe to get a neat end on it - I would expect that their next step is to fit such a valve. 87.102.43.94 (talk) 15:27, 5 June 2010 (UTC)[reply]
Update: our folks have fitted a new BOP just the other day, but we're keeping it partly open for now to keep it from getting jammed with clathrates. Right now only about 1/3 of the oil is being captured -- we'll be screwing it down to about 90% capture over the next few days, or to as low a flow rate as clathrate formation permits -- but we'll have to keep it open a crack until the relief well is completed. 67.170.215.166 (talk) 01:48, 6 June 2010 (UTC)[reply]
ok I'll bite.. if it gets jammed with clathrates does that not stop the oil flow - is that not a good thing?77.86.124.76 (talk) 14:59, 6 June 2010 (UTC)[reply]
You're wrong, 77 IP -- right now, the valve package is set up to divert part of the oil to a recovery ship, thus keeping the pressure down; if the thing gets clogged up with clathrates, it'll block that diversion and cause a pressure rise that could make the whole setup blow out again. 67.170.215.166 (talk) 00:58, 7 June 2010 (UTC)[reply]
Clathrates, as in clathrate hydrate and methane clathrate? Could that release greenhouse gases into the atmosphere? ~AH1(TCU) 01:36, 7 June 2010 (UTC)[reply]
This is not about so-called "greenhouse gases" -- it's about stopping the oil leak from killing all the fish in the Gulf and depriving the fishermen of their source of livelihood. We gotta keep our priorities straight. 67.170.215.166 (talk) 02:17, 8 June 2010 (UTC)[reply]

Nature/environment/biotope

Hi there! What's the difference between nature, natural environment and biotope? These three (especially the first two) concepts seem extremely similar but since there are three articles I suppose that there is some kind of difference. Could anyone help me to clarify this? /Natox (talk) 14:51, 5 June 2010 (UTC)[reply]

Nature and natural environment are almost synonyms. Nature is an older, looser term that refers to anything not man-made; natural environment is a bit more technical (though less technical than ecosystem or biosphere). Biotope is a particular type of natural environment. --Ludwigs2 15:17, 5 June 2010 (UTC)[reply]
(edit conflict - very similar answer)
Nature has multiple meanings (dictionary) one of which is synonymous (has the same meaning) as "natural enviroment".
As for "biotope" it usually refers to a specific type or area of natural enviroment - eg a "tropical biotope" or a "industrial wasteland biotope", it can even refer to small areas such as the types of biotope a bacteria might live in eg "gut biotope" , or even "fridge biotope" - the meaning and usage is quite similar in usage to ecosystem, but they are different and do have technically different meanings. ie an ecosystem will have many different biotopes within it - eg tropical rainforest ecosystem contains river biotope, stream biotope, stream bank biotope, leaf litter biotope, tree canopy biotope, mud biotope and many other biotopes.87.102.43.94 (talk) 15:20, 5 June 2010 (UTC)[reply]
Thank you both for good answers. This raises another question: How come there are two articles, natural environment and nature, if there is hardly any difference? They seem to be on the exact same subject. /Natox (talk) 15:39, 5 June 2010 (UTC)[reply]
Yes - I think that's a mistake - it looks like they should be (WP:MERGE) merged - at least I can't see any reason for two articles.87.102.43.94 (talk) 15:48, 5 June 2010 (UTC)[reply]
I've suggested that they be considered for merging.87.102.43.94 (talk) 15:54, 5 June 2010 (UTC)[reply]

Energy efficient of distillation

Assuming that any enthalpy change in the process is negligible, can the energy efficiency of distillation be arbitrarily close to 100%? Is there a reason why a certain minimum amount of energy will always be lost (i.e. become unavailable) in the process? --173.49.77.55 (talk) 16:14, 5 June 2010 (UTC)[reply]

(There needs to be an real valued enthalpy of vaporisation for distillation to be possible)
When you say efficient I assume you mean that 100% of the energy supplied to the system is converted into liquid boiling? If so yes it can approach 100% with effective insulation - there will always be a latent heat of the apparatus (ie start up) - ignoring this ie using continuous distillation the figure can approach 100% but not reach it because perfect insulation doesn't exist.
However if you say that the liquid for distillation is supplied 'cold' and do not include energy required to raise the T of the liquid (ie specific heat) then this will cause a less than 100% figure in the efficiency figure.87.102.43.94 (talk) 18:50, 5 June 2010 (UTC)[reply]
I was thinking about water purification by distillation when I asked the question. By negligible enthalpy change, I mean that you can ignore the energy that is inherently required to separate the impurities from the water, such as enthalpy of solvation. To simplify the problem, just consider the case when you're distilling pure water and condensing it back. Some of the energy used to boil the water can be recouped using a heat exchanger, such as a countercurrent heat exchanger. If the process is 100% energy efficient, the effluent water is at exactly the same temperature as the incoming water, and no incremental energy is needed to process an additional unit volume of water. That seems not possible in practice. Hence the question. --173.49.77.55 (talk) 23:46, 5 June 2010 (UTC)[reply]
I think the issue here is of getting energy to flow one way (from condenser to boiler). I think what you are describing is (the same as) either using the boiling water as condenser fluid or connecting the boiling water and condensor fluid so that they are at the same temperature. Additionally for the 100% efficiency the whole apparatus would have to be insulated -this means that the receiver would be at 100C as well - this would make the whole apparatus in thermal equilibrium - so there would be no net liquid flow.
To get net flow the condensor would need to be at a lower temperature than the boiling liquid - for this a heat pump would be needed - I'm not sure whether this heat pump can be 100% efficient.87.102.43.94 (talk) 12:58, 6 June 2010 (UTC)[reply]
I think theorectically you are right, eg if the whole apparatus is at 100C and a heat pump (see heat pump and related topics) is used to move energy from the reciever to boiler then then theoretical efficiency is infinite.. But it still requires a finite energy input for anything to actually happen. Also what might need to be considered is the need for energy input to prevent a 50:50 distribution of liquid in receiver/boiler coming about be the two being in effective equilibrium.87.102.43.94 (talk) 13:06, 6 June 2010 (UTC)[reply]
start again simple explanation - The idea of the distillation is to transfer a liquid from A to B , both at the same temperature. The energy required to do this is 0. But to actually get anything to happen there needs to be a non-zero energy difference between the two states (start state and finish state) - a non zero amount of energy will need to be supplied constantly to maintaine an energy differential (A>B) while the process is proceeding.87.102.43.94 (talk) 13:58, 6 June 2010 (UTC)[reply]
The energy required can approach zero, but when it is zero nothing happens.87.102.43.94 (talk) 14:00, 6 June 2010 (UTC)[reply]
If the purpose of the destillation is to desalinate water, you must take into account that the vapor pressure of salt water is lower than the vapor pressure of sweet water, so without doing work on the system the process will go in the wrong direction. The salt water must be warmer in order to increase to vapor pressure above that of sweet water. Bo Jacoby (talk) 21:29, 6 June 2010 (UTC).[reply]

Beer question

Why does beer taste better out of a glass bottle than a can? I'm mainly talking about bitters and ales here, I don't tend to drink many macrobrew lagers. 188.221.55.165 (talk) 17:48, 5 June 2010 (UTC)[reply]

Glass doesn't taint the liquid inside it. I'm sure someone will contradict me, but even if cans are lined with plastic, you can taste the tin, which must mean it has tainted the contents. Also you can't put real ale into a can because it will have to be pasteurised during the canning process, which means it loses its real ale status. Filtering and pasteurising beer will also alter the taste. You can get real ale in a bottle, and you can have filtered and pasteurised ale in a bottle. Personally (and I will be drummed out of CAMRA for saying this) I will gladly drink a bottle of beer even if it's not real ale, because the quality can be almost guaranteed. --TammyMoet (talk) 19:17, 5 June 2010 (UTC)[reply]
For one thing, it's often cheap beers that are distributed in cans. To make a fair comparison, you'd need to look at the same brand of beer in both cans and bottles. From a manufacturing standpoint, canned beers are clearly superior: it's easy to seal a can, and no light can get in. I would suspect that both the glass and can are relatively neutral in regards to taste, and that it's much more likely to be the feel of glass vs. metal that makes a difference in terms of enjoyment (and note that beer cans are aluminum, not tin, as TammyMoet incorrectly asserts). Many of the people online suggest that beer should always be drunk from a (glass) cup, whether it comes in a can or a bottle. See also [21] for one among many opinions on the matter. Googling "beer can vs bottle" will bring up tons of other opinions on the matter. Buddy431 (talk) 22:15, 5 June 2010 (UTC)[reply]
'Tin' is used as a synonym for 'can' in the UK. I doubt Tammy is asserting anything about the composition of the can/tin, only that it can be tasted. And indeed it can, although I suspect that is mostly the smell from the metal on top, particularly the exposed surface you've just created in opening it. I wonder if the same effect is evident when the beer is decanted before drinking. 86.164.69.239 (talk) 22:48, 5 June 2010 (UTC)[reply]
Agreed to the small comment above; beer in a can is different to beer in a bottle. Try Budweiser in a beer can and bottle, you'll notice a slight difference. The tin taints the beer slightly. Regards, --—Cyclonenim | Chat  23:29, 5 June 2010 (UTC)[reply]
BTW, there is no tin in tin cans. They are made of steel or aluminum. And for a proper test pour the beer into a glass before drinking it. Ariel. (talk) 03:37, 6 June 2010 (UTC)[reply]
To clarify, I wasn't asserting cans were made out of tin, but in the UK and associated countries, cans are known as "tins". When an Australian refers to a "tinnie", she is referring to a can of beer. --TammyMoet (talk) 08:15, 6 June 2010 (UTC)[reply]
The UK/Aussie usage is based on the fact that cans were originally lined with tin, and were called tin cans. Often, the "can" was just dropped, and hence the popular usage of just calling any metal container a "tin". --Phil Holmes (talk) 11:54, 6 June 2010 (UTC)[reply]
Not all tumblers are made of glass; some are made of plastic. -- Wavelength (talk) 15:21, 7 June 2010 (UTC)[reply]
The OP needs to first test the assumption by doing a blind taste test among several friends. Its an easy thing to do: Find friends that claim they can tell the difference between bottled and canned beer, obtain cans and bottles of the same brand, then pour each into identical glasses, and see who can tell the difference. --Jayron32 03:14, 7 June 2010 (UTC)[reply]
I always assumed that it was actually DRINKING it out of the can that made the difference, if you poured it into a glass it would taste the same as the stuff from a bottle... Also, wouldn't a can be similar to the kegs that pubs use? I don't know of anyone complaining that beer from a tap tasted "tinny", i thought keg beer was sort of the "gold standard" as long as the pub cleans and maintains their pipework. Vespine (talk) 06:28, 7 June 2010 (UTC)[reply]
The "gold standard" is real ale out of a wooden barrel! Keg beer is chilled, filtered and pasteurised to remove anything that could possibly give rise to any taste! Keg beer does indeed taste tinny. (Sorry I know I've just got on a soapbox, but a statement such as Vespine made deserve correction.)--TammyMoet (talk) 15:11, 7 June 2010 (UTC)[reply]
You don't get "tinny" kegs, but you do get dirty ones! Even worse. Regards, --—Cyclonenim | Chat  23:10, 7 June 2010 (UTC)[reply]
Right sorry i should have qualified, i am a heathen when it comes to beer! I avoid those ultra filtered beers and i drink mostly bottled brewed ales, coopers pale ale is my favorite, but I am FAR from a connoisseur. Having said that, i have never drunk, or even heard of anyone ever drinking beer from a wooden beer barrel! If you're going for a pint of Guiness, you go to the pub to get it from the tap (hence keg, i'm guessing), because that's where it's the best, is how I understood it, if the alternative is in a can or bottle. Not even the specialist microbreweries I have tried have beer in wooden barrels, so sorry but i have no idea what you're talking about there lol, you must be a bigger beer snob then anyone I've met, no offense. ;) Vespine (talk) 03:54, 8 June 2010 (UTC)[reply]

Vegetable cooking oils - two groups of questions

I never fry food but I do drizzel pasta with a small amount of cold oil. Questions 1) Trans-fat in oil. This http://www.nal.usda.gov/fnic/foodcomp/search/index.html says that canola/rapeseed oil is 0.395% trans-fat, and sunflower is 0.219% trans-fat. It does not give any figures for the other oils I have looked up. How or why is the trans-fat in there? Where can I find reliable figures for the amount of trans-fat in other vegetable oils?

Question 2). I have just bought a litre of "vegetable oil" from a supermarket in the UK. What is it likely to be, more specifically? Thanks 92.24.182.48 (talk) 18:41, 5 June 2010 (UTC)[reply]

(2) It's likely to be rape oil. 213.122.16.179 (talk) 19:28, 5 June 2010 (UTC)[reply]
agree - there's a list at Cooking_oil#Types_of_oils_and_their_characteristics some of which are not vegetable, some of which are too flavoured to be used, but quite a few can and are used in a vegatable oil.
There's a more specific list [[22]] it's quite likely to be a mixture as well.87.102.43.94 (talk) 20:26, 5 June 2010 (UTC)[reply]
Note that Canola oil is merely a type of rape oil. I think that "vegetable oil" is likely to be a mixture consisting of whatever's cheapest/easiest to obtain at the time of manufacture. Buddy431 (talk) 22:08, 5 June 2010 (UTC)[reply]
Well, it's not "merely a type" of rapeseed oil — it's the only type, as far as I know, that's considered fit for human consumption. Rapeseed oil other than Canola has too much erucic acid and is considered toxic (maybe not very toxic, but it's not a foodstuff). So I kind of doubt it would be any other sort of rapeseed oil.
Unless "vegetable oil" in the UK doesn't have to be fit for human consumption? Is it something you'd use for, I don't know, thinning paint or something? --Trovatore (talk) 09:29, 6 June 2010 (UTC)[reply]
Indeed. In the U.S. "vegetable oil" tends to be primarily soybean oil, or a canola/soybean blend. Pure canola oil (labeled as such) tends to be slightly more expensive. I've also seen "vegetable oil" where the ingredient list was "one or more of soybean, corn or canola oil" (or something similar) implying that the mixture varied based on what was available cheap the day they bottled it. -- 140.142.20.229 (talk) 22:31, 5 June 2010 (UTC)[reply]
I have two bottles of vegetable oil in my kitchen - one Morrison's own brand, the other Lidl's own brand. Both are labelled as "vegetable oil", and both are 100% rapeseed oil. "Canola" is not a British English word. DuncanHill (talk) 23:47, 6 June 2010 (UTC)[reply]
(2) About 3 years ago I saw a television documentary about the vegetable oils used in food. They claimed that the cheapest vegetable oil is the palm oil. They also claimed that that was a bad thing because palm oil supposedly is far more likely to cause you diseases in the heart and blood vessels (cardiovascular disease) than any of the other vegetable oils. They even went as far as to point out (and warn against eating) specific food products who are made of palm oil, and who just say "vegetable oil" on the label.
--Seren-dipper (talk) 23:10, 6 June 2010 (UTC)[reply]
Unfortunately I do not have any references to eventual scientific research that would back up the following claim, and I do not remember any of the explanation about why, but I have it from a third hand source (a non scientific worker at a food safety authority) who claims she heard it at her workplace that the healthiest oil to sprinkle on your salad is sunflower seed oil, while the healtiest oil (the one that will turn least harmful) when exposed to high temperatures (i.e. fried or baked) is rape oil.
--Seren-dipper (talk) 23:39, 6 June 2010 (UTC)[reply]
I believe that the consensus now is that canola/rapeseed/rape oil (worldwide naming conventions vary) is the best, due to having high amounts of monounsaturated fats and comparatively high amounts of Omega3. The data from the USDA link above gives - Sunflower oil: saturated 9%, monounsaturated 57%, polyunsaturated 29%, trans 0.2%, Omega3 0.04%. Canola oil: saturated 7%, monounsaturated 63%, polyunsaturated 28%, trans 0.4%, Omega3 9%. Thus sunflower has more 'bad fats' (saturated) and less 'good fats' (monounsaturated, Omega3) than canola oils. The details given in the sunflower oil article are for high oleic (70% and over) sunflower oil, which I am doubtful is what you get when you buy sunflower oil in a supermarket. 92.24.182.231 (talk) 09:51, 7 June 2010 (UTC)[reply]
That palm oil contains more saturated fats that sunflower oil etc seems to be the health issue eg [23] To complicate things there is an ongoing war in the literature - you find researchers from nigeria etc (places producing palm oil) pushin the health benefits and researchers from places that produce soya, sunflower oil etc saying it's not healthy.77.86.124.76 (talk) 00:15, 7 June 2010 (UTC)[reply]
Additionally there are different types of palm oil - with quite different constitutions [24] 77.86.124.76 (talk) 00:17, 7 June 2010 (UTC)[reply]

Standard Atmosphere...

This is a follow on from the highly informative 'boiling water' disscusion..

i) ' What is a standard 'atmosphere'  ?'

My very rough answer :
' A standard atmosphere, is a precisely determined combination of gases, which given that combination of gases has specfic definable physical properties, and obeys defined relationships between those propeties.'

However, this got me thinking, hence the following question :

ii) What factors need to be considered that might make an atmosphere non-standard

To which my initial thoughts where -

  • humidity (ie. 'humid' vs dry atmosphere) - and the 'liquid' component need not be water of course ;)
  • density of gas present.
  • liquid in proximity.
  • 'dust'
  • ionisation ?

Any thoughts from the Desk contributors? Sfan00 IMG (talk) 20:21, 5 June 2010 (UTC)[reply]

For physical measurements of bp the only factor that seems to be considered is pressure. see Standard conditions for temperature and pressure also Atmospheric_pressure#Standard_atmospheric_pressure and Atmosphere (unit) (some duplication of information)
As to definining what "standard air" is there's some links at Standard atmosphere, both U.S. Standard Atmosphere and International Standard Atmosphere are relevant.
In fact "one atmosphere" has been defined as both 100000Pa and 101325Pa (see first link)
The IUPAC doesn't seem to specify any other conditions other than pressure... There's definately at least two areas of definition - one for physics and another for atmospheric science.87.102.43.94 (talk) 20:38, 5 June 2010 (UTC)[reply]
The term I recall from high-school chemistry is not standard atmosphere (an atmosphere is a unit of pressure; no need for the standard), but standard temperature and pressure, abbreviated STP. If I recall correctly, STP was 25 degrees C and one atmosphere. Whereas my dad's chemistry books all seemed to refer things to 20 degrees C. I think I prefer that way. I find 25 Celsius (77 Fahrenheit) to be a little warm. -Trovatore (talk) 20:31, 6 June 2010 (UTC)[reply]

tobacco

during some wars they would drink pinecons because of coffee shortages. what did they smoke when there were tobacco shortages. —Preceding unsigned comment added by Alexsmith44 (talkcontribs) 21:16, 5 June 2010 (UTC)[reply]

According to our (fairly obscure) article Military_Administration_in_Belgium_and_North_France, sunflowers and jerusalem artichokes were used in the 40s. Nicotine, however, isn't present in either of those, so they would probably be rather unsatisfying. I tried to dig around on Google but couldn't find anything. When ersatz products weren't needed, cigarettes were frequently rationed. The Imperial War Museum has a picture of a tobacco tin containing tobacco substitute from the end of the first world war, although it doesn't describe its composition. Brammers (talk/c) 22:41, 5 June 2010 (UTC)[reply]
Hmmm. This rings a bell. I BELIEVE that there were certain plants (though I don't remember what they are) which would be -mixed- with tobacco, even in the smallest quantities. So you could have a cigarette with a small about of tobacco, and something else to bulk it out. I guess this gives you a little bit of the taste and nicotine. Hmmmm. SmokingNewton (MESSAGE ME) 00:59, 6 June 2010 (UTC)[reply]
A chap I knew who was in the navy in WWII said they smoked tea leaves when the tobacco ran out on board ship. He also said that this wasn't "very nice" but it was all they had. He told me that tea was always in abundance.--TrogWoolley (talk) 14:12, 6 June 2010 (UTC)[reply]
In the Second World War, British forces were issued with "Victory V" cigarettes. They were generally believed (because of their distinctive taste) to be made from a camel by-product. Some links here. DuncanHill (talk) 14:23, 6 June 2010 (UTC)[reply]

June 6

Can the Deepwater Horizon oil spill slow or even stop the ocean conveyor?

I realise that the complexities of the ocean conveyor are incredibly complex, and that any answers to this may be guesses at best. I read "Shutdown of thermohaline circulation" which varies from 5000 years (citatation needed) to just 2-3 years (especially from ice core data) for the Earth to swap to a full Ice Age. Scientific experiments currently point to a dramatic and exponential slowing of the conveyor, but again we don't know enough to say, "Well going by this in 7 years it will have stopped and we'll be in Ice Age".

But we do know that salinity in the water seems to drive it, and it's a very finely balanced system, which has numerous times in the past swapped state.

Given that we seem to be in the middle of such a swap, could the enormous amount of oil (which continues to rise with no current idea of how to stop it) stop the ocean conveyor? If it is so finely balanced and mere salinity of the water is extremely important, then wouldn't a Gulf full of crude oil be enough to stop the whole thing quickly? Could we be in ice age in 2010? I'll be very happy for any experts to shoot that idea down ..

blucat David —Preceding unsigned comment added by 198.142.19.12 (talk) 00:37, 6 June 2010 (UTC)[reply]

Prob'ly not -- the oil slick (as bad as it is) only covers a fairly small part of the Gulf's surface, so its effect on salinity is pretty insignificant. Also keep in mind that a large part of the oil is in underwater plumes, which have no effect on evaporation. Of course this is just a non-expert opinion from a reg'lar schmo who works at a refinery while taking evening classes at the university and writing adventure novels on weekends, and who knows a lot about oil refining but not much about ocean currents. FWiW 67.170.215.166 (talk) 01:57, 6 June 2010 (UTC)[reply]
The volume of the "enormous amount of oil spilled" in the "gulf full of crude oil" at the latest high limit for the leak rate of 19,000 barrels/day (USGS)is about what percent of the volume of water in the gulf, where a cubic meter=about 6.3 barrels of oil? I get about .000000006%. Edison (talk) 03:12, 6 June 2010 (UTC)[reply]

Plugging holes in ducts

I have a duct which conveys air under pressure. I have to create some holes in it for measurement. The holes will be larger than conventional ones - maybe 10 - 25mm in diameter. Can anyone suggest some easy ways of plugging them after I have finished? Thanks —Preceding unsigned comment added by 122.175.80.243 (talk) 04:27, 6 June 2010 (UTC)[reply]

What kind of pressure are we talking about? If it's fairly light pressure, duct tape on the inside of the duct might be easiest (maybe lain over two-inch cardboard or plastic circles for a bit of stiffness). duct tape around the outside might be sufficient as well, but it's ugly and not as strong. --Ludwigs2 04:44, 6 June 2010 (UTC)[reply]
As Ludwigs2 said we will need to know what sort of pressure the air is under before we can offer sensible answers, as depending on the pressure solutions will range from duct tape on the outside as mentioned above to things involving welding and steel plates. Another thing - is your only access to the inside of the duct through the holes, or can you reach the inside of the holes to e.g. apply patches? Equisetum (talk | email | contributions) 11:08, 6 June 2010 (UTC)[reply]
Duct tape can eventually fail. Metal tape with mastic like is used around flat roofs will deal with most problems with air conditioning ducts. For something this small just gluing something on it might do the job. Dmcq (talk) 12:35, 6 June 2010 (UTC)[reply]
Speed tape. In the UK I think it's just called "aluminium tape" (it's not just coated with aluminium, there's a Al foil as well) - it's definately more suitable for ducting that 'duck tape' especially in terms of gas resistance.77.86.124.76 (talk) 15:15, 6 June 2010 (UTC)[reply]
Corks or bungs? Works for champagne. 92.24.185.225 (talk) 13:48, 6 June 2010 (UTC)[reply]

This is for a pneumatic conveying system so pressure will be fairly large (~8 kPa). It is a negative pressure (suction) so I guess taping (IF that is possible) would be done on the outside (right?). No the only access to the inside is through the holes. Any convenient method to plug it? Thanks —Preceding unsigned comment added by 122.175.80.243 (talk) 14:32, 7 June 2010 (UTC)[reply]

The ultimate fix would be to weld or glue a cylindrical or semi-cylindrical patch over the hole. This could be made of the same material as the tube itself. Using a large enough length of 'patch' would give you plenty of contact area to get an airtight seal over. SteveBaker (talk) 19:07, 7 June 2010 (UTC)[reply]
The ULTIMATE ultimate fix would be a blow-out preventer. LOL 67.170.215.166 (talk) 02:19, 8 June 2010 (UTC)[reply]

Lee Smollin: author of "baby universe" book, adversary of multiverse? huh?

Lee Smolin wrote a book advocating (or maybe just describing the theory of?) "baby universes." Years later, in a celebrated comment, he said "This universe is the only one there is." I can't figure out if this means he abandoned the baby universe theory altogether, or if he was defining "universe" to mean "the universe that contains all the baby universes." Does anybody know how Smolin's two positions relate to each other? 63.17.74.45 (talk) 08:38, 6 June 2010 (UTC)[reply]

I don't, but note that one was written and another spoken, there's often quite a difference between what people are prepared to say and what they will write, my guess the the book was hypothesising.87.102.43.94 (talk) 13:39, 6 June 2010 (UTC)[reply]
They're both in print, the latter in this article (requires a login which you can get here). He evidently means "the universe that contains all the baby universes", making it a rather content-free statement as far as I can tell (though he seems to think otherwise). -- BenRG (talk) 19:32, 6 June 2010 (UTC)[reply]

BBC Report

I heard yesterday on the BBC Radio World Service that I was listening to while playing World of Warcraft, that the Cassini probe has discovered what could be life on Titan, due to certain gasses in the atmosphere being absorbed, or not reaching the serfice (surfice, sorry I have gone blank on the spelling???) This was anounced by NASA. I would like to read our article on this if we have one, can some one please link to it. Also, I have ready the article on Terraforming planets, and would like to read more in this field, can you please sugest articles in this vein. What university courses could one take to help further the future science of Terraforming, to help bring about this phenomenon faster, as I would love to be the man who 500 years from now is known as the father of Terraforming, or the Father of Human Colony II Mars. Thank you. —Preceding unsigned comment added by 62.172.58.82 (talk) 10:17, 6 June 2010 (UTC)[reply]

The featured article on Titan contains a lot of information, including cross-refs to the Cassini–Huygens mission and a section on Prebiotic conditions and possible life. We also have Colonization of Titan, and this article, which states that: "A widely published expert on terraforming, Christopher McKay, is also a co-investigator on the Huygens probe that landed on Titan in January 2005." Hope this helps - and good luck in getting there! Ghmyrtle (talk) 10:27, 6 June 2010 (UTC)[reply]
This is the story reported on the BBC: Hints of life found on Saturn moon --TammyMoet (talk) 10:44, 6 June 2010 (UTC)[reply]

Space travel

Given a timeframe of 5-10 years to create a vehicle, what is a reasonable quote for the amount of time it would take to send a probe to another solar system, given current technological levels? Ideally, the vehicle would return information to the earth about the solar system. I am assuming that current technology levels would not be good enough to reach relativistic speeds, so the perception in difference in time between the vehicle itself and the earth would be negligible (i.e., well within the margin of error for the quote). Magog the Ogre (talk) 16:14, 6 June 2010 (UTC)[reply]

Around 60,000 years. (I figure Cassini is currently at 25km/s, and 5 light years distance, is 60,000 years.) Cassini will slow down as it gets farther from the sun, but we could make some future ship a bit faster. But realistically we couldn't do it. The problem is how to decelerate when you reach the end point. We don't even know what the solar system looks like, we'd need artificial intelligence good enough to survey planets, then pick and calculate a gravity slow down, I'm not sure we are able to do that. Ariel. (talk) 17:10, 6 June 2010 (UTC)[reply]
That's hideous. Is there no manmade technology that could be put into a probe to quicken that span? Magog the Ogre (talk) 20:12, 6 June 2010 (UTC)[reply]
I expect the best method (using already existing technology) would be a kind of ion drive. The advantage of ion drives is that they can continue thrusting for a long time. If we want to launch in the next 10 years, though, it's still going to take at least hundreds of years. It would get there sooner if we waited until faster technology was invented and then launched. --Tango (talk) 20:37, 6 June 2010 (UTC)[reply]
There is no need to decelerate. It would be nice to do so, but you could still get lots of good science from a fly-by mission. I would expect our first interstellar missions to be fly-bys (just as our first interplanetary missions were). --Tango (talk) 20:37, 6 June 2010 (UTC)[reply]
I believe that if cost where not an issue, we could get a probe to the Alpha Centauri system in "just" a few hundred years. Ion thruster systems get better every year, (this report shows no hint of failure after more than 3 years at full power), and given their great mass/force ratio, they would be ideal for such a mission. Unfortunately I can't find the specs on the thrust they are capable of (which are quite small in comparison to chemical rockets), but even assuming just 10 km/s/year, (about 1/1000 the acceleration due to earth's gravity), the probe could achieve speeds more than 4 times that of the fastest current probes before it left the Solar System.-RunningOnBrains(talk) 21:01, 6 June 2010 (UTC)[reply]
It would be tough to power it. Assuming you ran for about 10 years, and the ship weighed 2 tons, you would need 10x10^12 joules, or about 32kW for those 10 years. Radioisotope thermoelectric generators have energies in the tens of watts, so that will never work. You would probably need a nuclear reactor such as Safe Affordable Fission Engine. But even then it would still take 15,000 years. Ariel. (talk) 01:41, 7 June 2010 (UTC)[reply]
In fact, I believe the biggest problem with this mission would not be getting there, but relaying what the probe sees back to earth. It would be useless to send a probe there if it can't send any information back, and New Horizons, after its fly-by of Pluto, is expected to take nine months to transmit just 8 GB of data back to Earth [25]. The amount of power needed to even reach those data transfer rates from 4 light years (and near the shine of a bright star)...well, I don't even want to begin to speculate.-RunningOnBrains(talk) 21:01, 6 June 2010 (UTC)[reply]
Let's speculate. New_Horizons#Telecommunications quotes 12W transmit power and 4.5 'light-hours' (signal latency). Link power is proportional to distance squared. Link power from Alpha Centauri would be 12W x sqr( 4.4 lightyears / 4.5 lighthours) = 1.1 kW Cuddlyable3 (talk) 21:38, 6 June 2010 (UTC)[reply]
A year is about 8000 hours, so multiplying by the square of the ratio (year/hr) should give you something like 700 MW, not 1.1 kW. Maybe you took the square root instead of the square? --Trovatore (talk) 21:50, 6 June 2010 (UTC)[reply]
Google says: 882 MW. 1.1kW is, indeed, what you get with a square root, not a square. --Tango (talk) 21:59, 6 June 2010 (UTC)[reply]
It occurs to me that one way to do better is to make the transmitting antenna bigger. With a bigger antenna you can get a more unidirectional signal — ultimately the power is still going to fall off as the square of the distance, but you can make the constant coefficient much more favorable.
How about having it shoot out some sort of very fine webbing a kilometer across, and using that as an antenna? Maybe it could even double as a light-sail. Obviously there are some practical challenges, but that's engineering's department; I'm over here in blue-sky big-picture territory. --Trovatore (talk) 22:10, 6 June 2010 (UTC)[reply]
The alternative is to use a laser. --Tango (talk) 22:30, 6 June 2010 (UTC)[reply]
Effectively that's like using a bigger antenna, because the important thing is the ratio of the aperture size to the wavelength (assuming spatial coherence across the entire aperture). But sure, a laser might be more practical than my idea. On the other hand it can't double as a light sail. So put together a presentation and present it at next week's meeting, please. --Trovatore (talk) 23:00, 6 June 2010 (UTC)[reply]
The Project Orion spaceship project was designed to reach 10% the speed of light, before it was scrapped. This would allow it, assuming constant maximum speed, to reach the Alpha Centauri system in 45 years. ~AH1(TCU) 01:27, 7 June 2010 (UTC)[reply]
Apparently we've all forgotten that NASA is actively testing a solar sail, which can propel to relativistic speeds without needing internal power (simply getting momentum from sunlight. Of course, a solar sail ship needs a sail on the order of 1 km^2 in size and extremely lightweight, but technologically we're already capable. Oh, and if you want, it can decelerate for free as it approaches a new solar system, though relativistically that takes quite a long time in the ship's reference frame. SamuelRiv (talk) 05:35, 7 June 2010 (UTC)[reply]
Not only NASA. -84user (talk) 01:03, 8 June 2010 (UTC)[reply]
Assuming you've got something wizzing very fast towards a star as described above, would it be possible to put it in orbit around that star? If it was going very fast, I imagine it would have to pass close to the star to be captured in orbit around it. Would this mean that it would be destroyed by the heat or gravity or other forces 92.24.182.231 (talk) 10:29, 7 June 2010 (UTC)[reply]
I would refer to the article on orbits. With escape-velocity speed, an object will travel around a star as a hyperbola, thus never returning. It may be that to get into elliptical orbit one has to get so close to a star as to be within the star itself, which is problematic because in doing so the star's gravitational influence on the object is reduced (see shell theorem), so the elliptical orbit is impossible anyway. But even if an elliptical orbit is established, at very high speeds the result will be a very eccentric orbit going very very far from the star and lasting perhaps thousands of years or more, as in Oort cloud objects. SamuelRiv (talk) 15:27, 7 June 2010 (UTC)[reply]
If the target solar system had some handy gas giants (which is looking like a pretty safe bet) - then you could probably do some aero-braking manouvers to bring your craft's speed down to where an orbital capture would be feasible. But the number of times you'd have to loop around to do that might be kinda large...it might take decades to get yourself into a comfortable orbit. SteveBaker (talk) 19:04, 7 June 2010 (UTC)[reply]

How can using algae to make fuel be net energy positive?

Recently on the Science Channel some people like J. Craig Venter and Ray Kurzweil were talking about the prospects for the future of genetic engineering and one of them (I forget which one) was talking about making life that would more efficiently manufacture fuels (by which I presume he meant hydrocarbons) humans could use. The problem I don't get is, if you want a bacteria to produce for you a molecule of hydrocarbon that could release X joules of energy when you use it, wouldn't you have to feed that bacteria more than X joules to make it for you? So if you have X+ joules it would be more logical to store that up than use for your energy needs. 71.161.47.112 (talk) 16:55, 6 June 2010 (UTC)[reply]

See algae fuel - algae make fuel from sunlight - but different algaes make different types of fuel: some primarily carbohydrate/cellulose, others produce a substantial amount of oil as well. I'd guess they were suggesting making new types of algae that produce more oil as a percentage, since this is a more desirable form of fuel.77.86.124.76 (talk) 17:03, 6 June 2010 (UTC)[reply]
Algae are not bacteria (except blue-green algae which are, and should be called cyanobacteria), they are a lot like plants.77.86.124.76 (talk) 17:08, 6 June 2010 (UTC)[reply]
Comparing the first paragraph of Photosynthetic efficiency which says it's about 3-6% to http://en.wikipedia.org/wiki/File:PVeff%28rev100414%29.png , it seems like a battery with X joules charged by a solar cell is made off much less light than a container of oil of the same energy content made by algae. 71.161.47.112 (talk) 17:40, 6 June 2010 (UTC)[reply]
If we're talking about green algae then they usually are considered plants (not land plants obviously). Nil Einne (talk) 19:25, 6 June 2010 (UTC) [reply]
Yeah, but we're not talking about green algae; we're talking about blue-green algae, AKA cyanobacteria. Contrary to what 77.86.124.76 says, they are bacteria. However, in accordance with what 77.86.124.76 says, they are also "a lot like plants", in the sense that they use photosynthesis. --Trovatore (talk) 19:34, 6 June 2010 (UTC)[reply]
I tried to be careful in what I said (because I'm not a biologist and likely to make obvious schoolboy errors - as I did) - but all algae? Bacteria says they are unicellular, whereas Algae says it included multicellular forms. eg Are seaweed bacteria ?? please put me out of my misery77.86.124.76 (talk) 20:04, 6 June 2010 (UTC)[reply]
No, seaweed are not bacteria. But no one is proposing to make fuel out of seaweed. Well, as far as I know, anyway.
The problem is that the word algae comprises organisms that are more different among each other than you are from an earthworm. Green algae are plants; bluegreen algae are bacteria. You can't get too much more different than that.
Now, on reviewing the algae fuel article, it does look like I spoke a little hastily in assuming that all the fuel stock was to come from bluegreen algae. Apparently the fuel makers don't really care about the biology, just about getting the reduced carbon, so very small green algae are also OK with them. --Trovatore (talk) 20:11, 6 June 2010 (UTC)[reply]
It's ok I worked it out for myself, yes they use algae, in fact the article mentions as examples algae species, though cyanobacteria are potential biofuels too. [26] Anyway didn't you get the email - we're calling them cyanobacteria now to avoid just this confusion :) 77.86.124.76 (talk) 20:25, 6 June 2010 (UTC)[reply]
BTW, the perhaps key point is that whether bacteria or plants or whatever, photoautotrophs will be used Nil Einne (talk) 16:05, 7 June 2010 (UTC)[reply]

I reacted sodium acetate with yellow iron(III) chloride to form a blood-red solution. Is that iron(III) acetate? The article on iron(III) acetate says that it is insoluble, but in here it states that there is a coloration. My solution is soluble. Is it "nonbasic" iron(III) acetate as compared to basic iron(III) acetate, which is what the article is about? --Chemicalinterest (talk) 18:08, 6 June 2010 (UTC)[reply]

Yes. Boil it to get the oxy-acetate [27]
As for the article - it needs some corrective work - it definately should mention both acetates.77.86.124.76 (talk) 19:03, 6 June 2010 (UTC)[reply]
I took a picture of the acetate (
see right) and wanted to post it in, but it wasn't the right acetate that is described in the article. --Chemicalinterest (talk) 19:31, 6 June 2010 (UTC)[reply]
BTW, I did boil it and it fumed acetic acid and formed an insoluble orange-red chemical. --Chemicalinterest (talk) 19:33, 6 June 2010 (UTC)[reply]
I've tried to fix the article - it now mentions both compunds (Iron(III) acetate) 77.86.124.76 (talk) 19:58, 6 June 2010 (UTC)[reply]

chloroform mishap

I am trying to synthesis chloroform for a chemistry project and i ran into some trouble. the chemicals i combined were ethanol 80 proof calcium hypochlorate from pool tablets and a small amount of sodium bicarbonate to take care of the formic acid that should be made. anyways during the reaction i noticed prussa blue crystals forming at the bottom of the glass. any idea on what they could be? —Preceding unsigned comment added by 76.14.125.132 (talk) 22:02, 6 June 2010 (UTC)[reply]

I can't think of a reaction that would have made a blue colour off hand. Are you sure that the Calcium hypochlorite didn't have a blue additive in them - it seems some commercial formulations do eg [28] [29] 77.86.124.76 (talk) 23:52, 6 June 2010 (UTC)[reply]

Size of Rhinovirus

Hello, would anyone happen to know the size of a rhino virus (particularly the type which carries the common cold) ? I could not find anything on the Common Cold Article. Thanks! ~ QwerpQwertus ·_Talk_·_Contribs_· The Wiki Puzzle Piece Award 23:13, 6 June 2010 (UTC) [reply]

Approx 22-40nm, according to http://www.uq.edu.au/vdu/VDURhinovirus.htmHappymulletuk (talk) 23:23, 6 June 2010 (UTC) [reply]

Thanks! ~ QwerpQwertus ·_Talk_·_Contribs_· The Wiki Puzzle Piece Award 23:58, 6 June 2010 (UTC)[reply]

Private Moon base...

Let's assume I have a great amount of wealth (10's of billions of dollars). Is there anything to stop me contracting someone to build me a moon base?

What would the estimated cost of a self sufficient base be, including any systems needed to remain healthy on the surface for an adult lifetime (for example, detrimental effects due to reduced gravity).

Finally, (this depends on the answer), is there any individual or group with the funds to actually do this?

I look forward to your thoughts!!Happymulletuk (talk) 23:20, 6 June 2010 (UTC)[reply]

You would need hundreds of billions. Possibly even into the trillion dollar range to do that. Ariel. (talk) 23:43, 6 June 2010 (UTC)[reply]
Yes, such a thing would probably cost trillions (including matinence), though one may not even be possible with today's technology. ~ QwerpQwertus ·_Talk_·_Contribs_· The Wiki Puzzle Piece Award 00:04, 7 June 2010 (UTC)[reply]
"Self-sufficient" is probably beyond the range of current technology. To be truly self-sufficient, you'd need to be able to manufacture any spare part of any machine using only materials that can be mined from Moon rock. That's a very tall order. We might conceivably be able to extract water - and therefore oxygen and hydrogen+oxygen rocket fuel - but Moon rock points out that the only elements present in moon rock in appreciable quantities are Calcium, Aluminium, Silicon, Oxygen, Iron, Magnesium and Titanium. Without nitrogen or carbon (and a bunch of other stuff) - you are pretty fundamentally screwed when it comes to food production. You'd have to recycle every single gram of those two elements with fanatical attention to not losing anything whatever in the processing! You might manage to find deposits of carbon from meteorites that impacted the moon in the distant past - but nitrogen is harder.
As for funding, Forbes list of billionaires says that no single person could fund more than about $55 billion - and that's nowhere near enough. Constellation_Program#Cost says that the simplest manned mission would cost in the region of $200 billion. What you are proposing (a permenant self-sufficient base) would probably cost trillions. A consortium of the top 20 richest people could possibly establish a moon base - but there is no way they could keep it funded over the long term. So, I think the answer is a solid "No". SteveBaker (talk) 00:49, 7 June 2010 (UTC)[reply]
The Outer Space Treaty appears to say that you have to get permission from your government first. The Moon Treaty says you can't own private extraterrestrial property at all, but fortunately only applies if you are in a country that ratified it, e.g. Australia or Belgium. (Don't those countries have any private satellites? Hm.) I have a feeling I once read that NOAA asserted a ban on anybody building anything on the moon (can't find any reference other than my own memory of the buzzkill). I'm not sure what would come of all this official indignation if you went ahead and built a moon base anyway, mind you. Maybe they'd blockade your space ports. 213.122.43.84 (talk) 04:39, 7 June 2010 (UTC)[reply]
Practically speaking though if you had ownership of such sums with complete discretion on how to use it you might well be able to "leverage" your money by bribing politicians to do it for you provided you were a bit subtle. I don't know how much it would cost to effectively control the US government NASA budget completely by campaign contributions but if you had $1bn a year to throw around I think you could probably get huge government expenditure in support of your aims... --BozMo talk 12:31, 7 June 2010 (UTC)[reply]

June 7

Avalanches

Can a strong gust of wind cause an avalanche? In particular, can an avalanche be triggered by the foehn winds that commonly occur in some mountain ranges (Alps, Rockies, etc.)? Thanks in advance! 67.170.215.166 (talk) 01:04, 7 June 2010 (UTC)[reply]

Since an avalanche can be triggered by a loud noise, it seems almost certain that they could be caused by strong winds -- but I can't see how it could ever be proven that any specific avalanche was caused by wind. Foehn winds tend to occur in specific weather conditions, so I don't see how you could disentangle the wind from the weather. Looie496 (talk) 01:22, 7 June 2010 (UTC)[reply]
Actually, avalanches caused by sound is an disproven fact (see Avalanche#Myths about avalanches). I have heard stories of low-flying helicopters causing avalanches, so I assume a strong windstorm could do it too. --The High Fin Sperm Whale 02:50, 7 June 2010 (UTC)[reply]
Well, triggers section of avalanches says they can be caused by warming, and the Foehn wind article says "Winds of this type are called "snow-eaters" for their ability to make snow melt or sublimate rapidly." so i think it's quite reasonable to suggest that they could cause avalanches. Vespine (talk) 06:36, 7 June 2010 (UTC)[reply]
At some european ski resorts, they use explosives to trigger controlled avalanches to make the snow safer for skiiers - so a sufficiently powerful shockwave will do it. Whether you call that "sound" or "explosive pressure" or "wind" is a matter of degree and nomenclature. A sufficiently powerful movement of air is enough. After all, in the few seconds before a 'natural' avalanche, almost zero disturbance is needed...and at the precise moment when the avalanche starts, zero disturbance was needed to trigger it. So there must be some balance point where a strong gust of wind could set it off...but that might be just minutes before the thing would have started all by itself. SteveBaker (talk) 13:13, 7 June 2010 (UTC)[reply]
I actually was going to say something very similar, but then i had a quick read of the avalanche article and it specifically says that they are not random or spontaneous events. So melting or pressure or shock can trigger it, so it might be as simple as the sun shining or a gust of warm wind. Of course it depends on what you mean by "all by it self" but I would avoid using that phrase, we're not talking about schrodinger's cat after all;). Vespine (talk) 03:45, 8 June 2010 (UTC)[reply]

Lithium phosphide cell

The lithium/lithium phosphide cell looks interesting (two different patents ):

Questions:

  • [30] would this cell produce phosphine if it got wet? and would that prevent it ever being used (eg any comparative examples of potentially dangerous substances contained in a product)?
  • [31] a conducting allotrope of phosphorus is used - this suggests black or purple phosphorus - if the cell is discharged would this make polymeric polyphosphide ions , and if so would these ions react with water to produce phosphine or not? or would it be intercalation instead?
    • also an Intercalation (chemistry) reaction known anywhere between either Li or Li+ and conducting phosphorus?
    • also in the cell described above - I'd like to know whether the conducting phosphorus would be expected to reduced when in operation - are any polyphosphide anions known? In fact is anything much at all known about the surface chemistry or modification of black phosphorus?

Oh and does anyone know of any more? 77.86.124.76 (talk) 02:03, 7 June 2010 (UTC)[reply]

While I'm partly guessing (although did glance thorough that section which seems to support my guess), I suspect the article only lists real world examples, in other words, chemistries that have actually been used to produce batteries used (at some time) in the real world (even if only in obscure/specialist areas or a few cases), rather then more theoretical examples, even ones that have been demostrated in the lab. However further discussion on this point would belong in the article talk page. BTW, one of those patents is from 1989. If there's no significant ongoing research, I would suspect that that proposal can be considered dead at the current time. Perhaps advances will change that but I would guess you'd see some published research when that happens. Nil Einne (talk) 17:11, 7 June 2010 (UTC)[reply]

Elder people

Why elder people easily feels cold while the environment temperature is normal for other adults? roscoe_x (talk) 03:07, 7 June 2010 (UTC)[reply]

Two reasons may be - poor circulation, and/or lack of physical activity. There are other reasons too I think.77.86.124.76 (talk) 03:10, 7 June 2010 (UTC)[reply]
The OP may be interested in reading gerontology and geriatrics, which discuss the issues related to aging from slightly different perspectives. --Jayron32 04:27, 7 June 2010 (UTC)[reply]
After the menopause, a woman's skin thickness naturally reduces and this could account for this phenomenon in women. ---TammyMoet (talk) 15:03, 7 June 2010 (UTC)[reply]

Neural basis of learning

What are the neural substrates in goal directed learning? (Neurobiology of action). —Preceding unsigned comment added by Oxio-a (talkcontribs) 04:15, 7 June 2010 (UTC)[reply]

There are dozens of different goal-directed and goal-oriented pedagogy programs. You're going to have to be a bit more specific. If you are interested in neural substrates in general, see the article Cognitive neuroscience, which discusses them. --Jayron32 04:25, 7 June 2010 (UTC)[reply]

Well, within the context of contingency learning and extinction. —Preceding unsigned comment added by Oxio-a (talkcontribs) 04:35, 7 June 2010 (UTC)[reply]

See Extinction (psychology) for a discussion of the phenomenon, as well as Operant conditioning for a discussion on that specific manner of learning. Again, your questions are a bit vague, so it is harder to provide more detailed responses than this. --Jayron32 05:13, 7 June 2010 (UTC)[reply]
I deal with a lot of computational and theoretical models of learning. The granddaddy of mathematical memory is Hebbian learning. One of the most promising real biological explanations is BCM Theory which is a calcium-based physics model for long-term potentiation. Hopfield nets are more abstract, and their biological relevance is extremely doubtful, but it does provide a neat simulation of learning, extrapolating, confusion, false memories, and extinction. In terms of goal-directed learning, there are self-organizing maps that work without a teacher and the standard adaptive algorithms that require training (like backpropagation). For the cells themselves, there is again LTP and STP (long-term and short-term) for two ways memory is encoded in the neurons (that is, how the neurons learn to rearrange themselves), accompanied by various models of how the synaptic "weights" change. I'm not sure where you're headed with your question, but I can give more details on at least some of these if you want. SamuelRiv (talk) 05:48, 7 June 2010 (UTC)[reply]
"Goal directed learning" is not a very meaningful term, but if you mean reward-driven learning, there has been a lot of work recently pointing to a central role for the basal ganglia, especially the projection from the ventral tegmental area to the nucleus accumbens that uses dopamine as neurotransmitter. Those articles should be enough to get you started. Many other brain areas come into play, but those are thought to be especially central to the neurobiology of action. Looie496 (talk) 05:56, 7 June 2010 (UTC)[reply]

CNG engines and simple engines

Could you please tell me the difference between engines of vehicles using CNG(Compressed Natural Gas) as fuel and those using petrol/diesel as fuel. If there is no difference then why not such bikes are made that can use CNG as fuel? And even if there is difference does this difference only made it costly or undesirable to use CNG as fuel in bikes?--Myownid420 (talk) 04:18, 7 June 2010 (UTC)[reply]
See Compressed natural gas which discusses advantages and disadvantages of using CNG as a fuel, and Natural gas vehicle which discusses some about the vehicles themselves. They operate on a variety of different designs, so you'd want to see List of natural gas vehicles and select various models to see how they operate, and how they differ from "standard" gasoline or diesel vehicles. --Jayron32 04:23, 7 June 2010 (UTC)[reply]
Well i don't think there is THAT much of a difference since it's not difficult to retrofit convert most petrol cars to LPG. Also, googling LPG motorbike it didn't take me long to find motorbikes converted to LPG. The obvious hurdle there is for LPG you typically don't remove the petrol tank and replace it, you just ADD an LPG tank, on a motoribke, there is obviously less space or convenience to add on a bulky LPG tank. As to maybe why there aren't bikes factory fitted with LPG to begin with, i suppose it's still just a matter of economics.. eventually petrol will get expensive enough that maybe bikes will get factory fitted LPG too.. Vespine (talk) 06:11, 7 June 2010 (UTC)[reply]
My father used to run a business converting gasoline cars to run "dual-fuel" on either gasoline or propane/LPG - so I have quite a bit of first-hand knowledge about this. The modifications to a car were as follows:
  1. A honking great pressurized gas cylinder - which would typically fill up most of the trunk of the car - but could be easily accommodated in the bed of a pickup truck or inside a larger commercial vehicle.
  2. A liquid propane line running from the tank into the engine's fuel intake - with a solenoid that switched between propane and gasoline, operated by a switch that was tucked away under the dashboard.
  3. A mechanism to route heat from the engine to the fuel inlet in order to prevent freezing as the propane expanded from a liquid into a gas. I think this took the form of a thin water pipe that ran from the radiator top-hose, around the fuel inlet and back into the radiator bottom-hose.
There are no changes whatever to the engine - you could certainly convert a motorbike to run on the stuff - and as a kind of sales pitch, my father converted a lawnmower to run on LPG. It's a very simple process.
The result was a car that had to be started on gasoline (because when the engine is cold, the carburettor would ice up if run on propane) - and could be switched over to propane within a couple of minutes of driving. If you were getting low on propane and there was no refilling station handy - you could simply switch back to gasoline at the touch of a button. You also needed to run about one tank of gasoline for every 10 tanks of LPG because gasoline contains engine cleaning agents and such that LPG doesn't have.
Liquid propane has a similar energy density to gasoline - so you got about the same MPG on propane or gasoline - but the propane was much cheaper, and (this was in the UK) didn't attract the killer taxes that are levied on gasoline there. Result: It cost you about 25% of the price per mile driven! A Taxi company that my father refitted with LPG systems paid for it in just 2 months of fuel savings. I don't know whether that's still the case though...and in places with lower gasoline taxes, the benefits are very much smaller.
The car produced only CO2 and water out of the exhaust - which was good for vehicles like busses that have to drive down city streets where people are walking. Of course this does NOTHING to help global warming - CO2 was still produced and in identical quantities to gasoline.
The problem with converting motorbikes is that of where to put the large, heavy gas cylinder. To get the same range, it would be the same volume as the bike's regular fuel tanks - but made of thick, heavy steel. You can't replace the existing gas tank because you have to run on gasoline until the engine gets hot...so you need both. So it's not a simple conversion. You could possibly design a bike from scratch to run on LPG with a really small gasoline tank with just enough fuel to get the engine started...but the inconvenience factor of having to fill a tiny little tank every few dozen engine starts would be a pain...and because LPG filling stations are not everywhere - you really need that ability to run on gasoline when the propane runs out. Also, unless the bike is water-cooled, you'd need to run exhaust gasses to the propane fuel inlet to prevent it from freezing.
LPG is widely used throughout the world for running fork-lift trucks. These machines need to be driven inside buildings where the build-up of carbon monoxide (not to mention the smell) from a gasoline engine would be unacceptable. Producing only CO2 and water means that these engines can be run indoors with zero problems for people in the building.
SteveBaker (talk) 12:55, 7 June 2010 (UTC)[reply]
"The car produced only CO2 and water out of the exhaust" only? c'mon Steve, surely the combustion fixed some nitrogen as well so NOx in exhaust, unless it really ran at low compression ratio? --BozMo talk 12:59, 7 June 2010 (UTC)[reply]
There was nitrogen of course - but no NOx. I guess that the combustion temperatures are lower or something. However, it's true - you could breathe what came out of the tailpipe. My father had trouble from the local planning authorities who complained that he operated motor vehicles indoors without exhaust capture systems - but he ran LPG cars in his garage all day with the doors shut. Again, note that fork-lift trucks are routinely run indoors on propane. SteveBaker (talk) 13:06, 7 June 2010 (UTC)[reply]

Eye Structure Of Toads

Hi! Could anyone describe the eye structure for a toad? Thanks! ~ QwerpQwertus ·_Talk_·_Contribs_· 05:10, 7 June 2010 (UTC) [reply]

All vertebrates have the same basic eye structure. Could you be a bit more specific about what you're trying to find out? Looie496 (talk) 06:04, 7 June 2010 (UTC)[reply]
Do they differ from a human's in any way, I guess? ~ QwerpQwertus ·_Talk_·_Contribs_· 07:00, 7 June 2010 (UTC)[reply]
They do not have binocular vision like humans, which enables humans to have finer depth perception and detail. Instead, they have bulging eyes which cover a lot of area; each eye looks at a separate area. --Chemicalinterest (talk) 10:50, 7 June 2010 (UTC)[reply]
There is a ton of literature on frog/toad and salamander eyes and retinas. What kind of difference from the human eye you are asking about specifically? Color vision? Anurans see colors, but not in the same L, M, S cone fundamentals as humans. Eye movement? They can lower the eyeball towards their throat and lift it back into the orbit; they have a pair of muscles for that. They can't make saccades like we do, though. Frogs don't really see non-moving objects unless the frog itself moves. Is that what you are asking? Ir is it something else? --Dr Dima (talk) 11:31, 7 June 2010 (UTC)[reply]
That's what I needed - Thanks! ~ QwerpQwertus ·_Talk_·_Contribs_·

gravitation

why the planets revolve around sun? and why they revolve around sun in an Elliptical path? is it due to the change in gravitation of sun? —Preceding unsigned comment added by 120.60.141.39 (talk) 05:42, 7 June 2010 (UTC)[reply]

Such a question was answered mathematically, with a final stamp, by Kepler's Laws. You're familiar with gravity - a big heavy object like the Sun has a big gravitational force that it pulls on the planets (as well as comets, asteroids, and anything else in our solar system). This gravity doesn't change like the mass of the Sun doesn't change, so why ellipses?
Let's say you have a big rock going really fast toward the sun (but just to the side) - as the rock gets closer, the Sun drags on it more and more, eventually swinging that rock around. But the rock is still going so fast that the Sun can't hold on and so that rock goes flying back out into space. BUT the Sun is still pulling, just much weaker, and the rock got slowed down when it went around the sun such that eventually the rock gets pulled to a stop and starts speeding up toward the Sun again, everything starting over.
That's a kind of intuitive description of an elliptical orbit. A perfect circle orbit can form, but it has to form kind of precisely, otherwise the planet will always have a moment where it will be moving faster than before and slip away a tiny bit before being pulled back. Note that these orbits are stable - that rock always comes right back where it started, speeding up toward the sun and flinging back away in the same paths every time. This is because there is no friction in space, so without the Sun changing or the rock changing, we get this great situation of Conservation of Energy, and you can do all sorts of nice math to prove all of Kepler's Laws from there. So the Sun doesn't change and gravity doesn't change, and that's precisely why orbits are elliptical. SamuelRiv (talk) 05:59, 7 June 2010 (UTC)[reply]
The strength of gravity follows the inverse-square law, as per Newton's law of universal gravitation. Thus if the planet were to double its distance from the sun, then the gravity between them would now be 1/4 of the original value. Strictly speaking, its the distance between the centres of the planet and the sun. Also the orbit is only stable if there are only the sun and the planet - other planets' gravity can affect the first planet's orbit. Even with one planet, the orbit's major axis will slowly rotate around the sun, see orbital precession for the details. CS Miller (talk) 12:54, 7 June 2010 (UTC)[reply]
I recommend our excellent article on orbits.--Shantavira|feed me 07:54, 7 June 2010 (UTC)[reply]

ph

whats the ph of 70 % nitric acid —Preceding unsigned comment added by Alexsmith44 (talkcontribs) 09:00, 7 June 2010 (UTC)[reply]

Very low. A 0.1M solution is pH 1.0, so a 70% solution would be close to 0, since it is a strong acid. --Chemicalinterest (talk) 10:53, 7 June 2010 (UTC)[reply]
it's -1, http://www.wolframalpha.com/input/?i=70%25+nitric+acid . The pKa of Nitric acid is -1.4, btw. --Rajah (talk) 16:55, 7 June 2010 (UTC)[reply]
Btw, wolframalpha gives -2 for Nitric acid's pKa, while wikipedia gives -1.4. Just an FYI. --Rajah (talk) 17:03, 7 June 2010 (UTC)[reply]
I think that at such extreme acidities, definitions become important, and I'm not even sure that everyone agrees on the definition. pH is supposed to be the negative base-10 log of the hydrogen ion activity, whatever that is, in moles per liter. The easiest thing to explain is the hydrogen ion concentration, and at low concentrations, that's (almost) the same thing as the activity. But when you get down to pH of zero or below, they may be rather different. I have never quite followed what "activity" means exactly, but I gather that it's some statistical-mechanics concept, and sometimes there are fuzzy aspects to some of those. --Trovatore (talk) 18:28, 7 June 2010 (UTC)[reply]


i dont know what 1m or pKa means —Preceding unsigned comment added by Alexsmith44 (talkcontribs) 23:22, 7 June 2010 (UTC)[reply]

See pKa and Molar (concentration). --Chemicalinterest (talk) 00:08, 8 June 2010 (UTC)[reply]

Twin Paradox - How will be the pulse count of the traveling twin?

Understanding Twin Paradox

The explanation that the difference of aging between the two twins is caused by acceleration or reversal of direction is not convincing if we compare two trips:

The voyager travels to a star distant 10 light years, starting with acceleration of 10 m/s till he attains 99% of the speed of light (what should take a little less than one year). He then proceeds with this speed until the same distance of about 1 light-year from the star and decelerates at 10 m/s2 until reaching a star at speed zero. Finally he returs to Earth in the reverse manner.

He then makes a second trip in the same way but to a star 20 light years from earth. The period and speed profile of the accelerations and decelerations will be the same in both cases, but in the second case the distance and time traveled at 99% of the speed of light would be about double.

If the age difference (and the respective clock markings) of the two twins were due to acceleration and / or direction reversal, the difference would be the same in the two cases.

Note: During the constant-speed step it is assumed that the ship rotates in order to generate a gravity of 10 m/s2 for the twin in order that he stays under normal living conditions.

Question: if the twin´s pulse rate were regular 60 per minute on Earth, how many beats he had accumulated at the main points of the travel? (start = 0, end of acceleration, start of deceleration, arrival at the star and the equivalent points at return.)

If the number of pulses of the traveler is much less than of his twin, then actually he has lived less during his trip. Also all chemical and biological reactions would follow this pattern. It is broadly similar to what happens to bull sperm kept in bottles immersed in liquid nitrogen: although its normal life is only a few days, it can be kept “alive” (actually in suspended animation) for more than an year because at this low temperature all chemical and biological reactions practically cease until the sperm is returned to normal temperature to revitalize and fertilize cows.

Question: If, starting with the ship´s departure, Earth sends a light signal every second, how many signals the ship has received at the main points indicated above? Also, if the ship sends a sequentially numbered signal every second (measured by the ship´s clock), at what time (earth clock) do these numbers arrive?

Question: IF the light signals from earth and from the ship are sent with a wave length of 600 nm, with which wave they will arrive at destination during the several steps of the voyage?

Question: in which reference frame (earth´s, ship´s, star´s) is the twin with his clock during acceleration and during constant speed, going towards star and going towards Earth?

In my opinion the problem of “understanding” the paradox as such does not exist because there is nothing in the universe what we really understand. We accept gravity, magnetism, electricity, matter, universe and life as facts and can treat many of them mathematically, but we do not really understand any of them. In the same way, what we need in the case of the twins is to know what happens, and this should be calculable by the formulae of restricted relativity and the Minkowski diagram.

07 June 2010 FJTREU 201.29.210.181 (talk) 10:21, 7 June 2010 (UTC)[reply]

Hmm, not sure so I won't answer. 86.4.183.90 (talk) 13:28, 7 June 2010 (UTC)[reply]
Your questions seem a bit loaded, and I get further every day from my physics degree, but I'll bite.
First, it is important to understand that the twin paradox is not a paradox at all; the problem and the system contained within are completely solvable with the known laws of physics. From your wording, I'm not sure if you understand this point.
Second, your assertion that the twin paradox is close in principle to Cryopreservation does not make any sense: one is physical time dilation due to relativity, and the other is merely the slowing/cessation of aging and decomposition processes due to extremely cold temperatures. These aren't even the same branch of physics.
Third, your calculation of taking around a year to accelerate to 99% of light speed is misleading. You are assuming that acceleration for a given force is linear—that the same force which accelerated you from 0 m/s to 10 m/s will get you from c - 20 m/s to c - 10 m/s.I'll leave the math to someone with more time on their hands, but if you wanted to have a truly constant acceleration, by the time you reached 99% of c, you would need inconceivable amounts of energy to get that last 10 m/s. But I digress...
Fourthly, you seem to misunderstand the very point of the whole paradox. Say all humans live exactly 1000 years (and, for the sake of your argument, let's make this a "triplet paradox", and assume that these are super-triplets whose bodies have all the same chemical reactions going on in exactly the same sequence). The first triplet is born, lives a fulfilling life, and dies exactly 1000 years later. The second triplet is born, then at age 20 goes on your 10-light-year trip. He comes back to earth, lives a fulfilling life, and dies. If he kept a clock, which is 100% accurate according to both his perception and the laws of physics in his reference frame, it will read that he died exactly 1000 years after he was born. Same goes for the triplet who went on the 20-light-year trip. Now, according to the triplet on earth, his siblings who went on the journeys will be much younger than him when they return, but this is only in his inertial reference frame. The cells in all three triplets bodies will undergo the same reactions and live the same length of time; this time will just seem different to someone who strayed from that person's inertial frame.
I hope that someone smarter and less busy will come along and answer your math questions, but they seem a bit trivial to understanding the problem as a whole.
And finally, well, your last paragraph is straying from the realm of science into the realm of philosophy. Science is not reality, it is a way of mathematically and quantitatively describing reality. "Accepting" science is a matter of personal choice I suppose, but the great thing about science is that anyone can do it—you can test and verify any law of science for yourself—and I assert that you will find the same values of the universal constants as everyone else (within experimental error of course). In my mind, I "understand" the science of magnetism, gravity, and relativity, but understanding science is easy, because science is the way we break down reality to make it easy to understand. -RunningOnBrains(talk) 14:08, 7 June 2010 (UTC)[reply]
Without doing the math, here's how to get out of the paradox. You state "If the age difference... of the two twins were due to acceleration and / or direction reversal", and that's the problem.
The observed age difference is because of travel at relativistic speeds. Period. Each twin observes time moving more slowly for the other while they are departing (because it's perfectly valid for each to claim that he's the one at rest). However, this creates a paradox -- if the twins are returned to each other, they can't each be older than the other one. The acceleration needed to stop and return resolves this. However, that's not the same as saying that the difference in time experienced is because of the acceleration. Note also, while philosophizing about guessing and the like, that this behavior has been experimentally confirmed. It's not guesswork. — Lomn 13:58, 7 June 2010 (UTC)[reply]
Careful! The observed age difference is due to the acceleration, since it is due to them traveling at relativistic speeds with respect to each other, when they begin in the same inertial frame. If they both go out at the same relativistic speed in the same direction and return, they will still be the same age. -RunningOnBrains(talk) 14:15, 7 June 2010 (UTC)[reply]
I guess the most effective (and thus appropriate) philosophy is the one given by Feynman: "Shut up and calculate!" The Twin paradox article shows a number of equations to assist people making calculations. DVdm (talk) 14:11, 7 June 2010 (UTC)[reply]
Or, as brilliantly put in one of the great underrated movies, "Do the math, motherf***er!". One interesting thing is that you can re-fold space (change its topology) such that the two ships actually can meet and synchronize without proper acceleration, but this violates global conservation of energy, which can occur when one changes topologies. Incidentally, this also occurs in wormholes, where the twin paradox is specifically exploited for time travel. SamuelRiv (talk) 15:19, 7 June 2010 (UTC)[reply]
Scheduled for this evening. TFTT :-) DVdm (talk) 15:45, 7 June 2010 (UTC)[reply]
Bummer :-( - DVdm (talk) 07:48, 8 June 2010 (UTC)[reply]

It's always amusing to see what people think "causes" the twin paradox, and how passionate they might be about their convictions. For the analogous Euclidean situation, what "causes" two distinct paths between Kathmandu and Istanbul to have different (or possibly the same) lengths? Is it the angles of deviation from a straight-line course? Maybe so, depends on how you want to look at it. Either way, no one seems to regard the fact that two paths between these cities don't generally have equal lengths as paradoxical. Go figure.

By the way, there's no need to invoke exotic topologies or energy conservation issues to have two ships with zero proper acceleration meet at different times. Two freely orbiting clocks in the influence of a common central force, one in a circular and one in an elliptical orbit, can repeatedly meet and show different elapsed times between meetings. Tim Shuba (talk) 21:05, 7 June 2010 (UTC)[reply]

Well, here's the disanalogy from your proposed Central Asian sojourn: A path from Kathmandu to Istanbul can be marked out along the ground, the physical structure, that is, of the Earth. You could even dig it into the soil and then measure the length later. Hard to argue with that.
The Twins, though, are making a journey across notionally empty space, and one of the basic driving ideas behind relativity is that there is no aether (or at least that, if there is, it's experimentally unobservable). So, one thinks, shouldn't we basing everything off of concrete objects? Well, what if the Twins and their spacecraft are the only objects there are? Where then does the asymmetry come from?
Now, it certainly could be "just the way things are" that there is no absolute frame of reference for position, or for its first derivative, but that there is one for the second derivative. But that definitely reduces the sense of a-priori canonicity with which the story of special relativity is usually told.
The other possibility is that you can just as well describe the events from the point of view of the Twin that we see as the one who went off and came back — but that to do so correctly, you have to include the acceleration of distant galaxies in the computation, and that when you do so, you wind up with the same observable outcomes. Here by "possibility" I just mean that I don't know this isn't possible — I'm no expert on GR and I don't know whether the equations really come out that way.
Once again, the relevant article is Mach's principle (whether that principle is true or false, I don't know — I'm not advocating it per se, just saying this is what we're talking about). --Trovatore (talk) 04:05, 8 June 2010 (UTC)[reply]
Although one twin may be "younger" than the other due to hyperrelativistic speed travel, cosmic radiation may cause that twin to age by damaging cell chromosomes enough to reverse the "de-aging" effect. ~AH1(TCU) 00:15, 8 June 2010 (UTC)[reply]

What determines the frame of referance for spinning?

The Solar sail article describes some designs of solar sail which would be kept stiff by the forces from spinning them. But why should the frame of refeance be the rest of the universe? If you took as your frame of referance the solar sail, then it would be static and the rest of the universe would be spinning around it. The solar sail should then go floppy. How can this be explained? How does "God" decide what the frame of reference should be in those circumstances? 92.24.182.231 (talk) 10:49, 7 June 2010 (UTC)[reply]

I don't think there's any broad agreement on this sort of question. You might be interested in Mach's principle, though. --Trovatore (talk) 10:52, 7 June 2010 (UTC)[reply]
You are confusing rotating frames of reference from those in linear motion relative to each other. Rotating frames of reference can be distinguished precisely because of effects like this. Frame of reference explains this. SteveBaker (talk) 11:48, 7 June 2010 (UTC)[reply]
Hmm? How am I confusing these? I simply pointed the original poster to the most relevant article. --Trovatore (talk) 18:21, 7 June 2010 (UTC)[reply]


The relevant article here is I think Frame-dragging. The rest of the universe spinning around would drag the local space round so overall you see no difference -the sails would still go out stiff. Dmcq (talk) 12:23, 7 June 2010 (UTC)[reply]
Is this an issue of Inertial frame of reference vs. non-inertial reference frame? 58.147.58.152 (talk) 14:53, 7 June 2010 (UTC)[reply]
Yes to above, people are getting confused. A rotating object creates a non-inertial reference frame, which is totally distinguishable from any inertial reference frames around it. The difference is that rotating objects have non-zero acceleration, so, as with relativity questions above, this makes all the difference and observers always know who is rotating and who is not. The sail is rotating because it feels centrifugal force. The universe is not rotating because, for example, humans on Earth do not (except from the Earth's own rotation). SamuelRiv (talk) 15:33, 7 June 2010 (UTC)[reply]
So ... if the sail were the only object in the universe - could you still tell whether it was spinning or not ? And if you fire rockets tangentially around the rim of the sail, will that make any difference ? What if you fire the rockets in the opoosite direction ? Now look at the sail from the other side - clockwise becomes anticlockwise and vice versa. So have you introduced a parity violation in an isotropic universe ? Gandalf61 (talk) 16:06, 7 June 2010 (UTC)[reply]
Uh? What parity violation? Dauto (talk) 16:33, 7 June 2010 (UTC)[reply]
In order for there to be an observer - the solar sail can't be the only object in the universe. If you and it were the only two things in the universe, you could certainly tell whether either it, or you, or both were rotating. However, this hasn't made the universe isotropic because everything depends on where the observer is relative to the sail. SteveBaker (talk) 18:56, 7 June 2010 (UTC)[reply]
But if the only two things in the universe were the sail and the observer, and they were rotating together, could you tell? Mach's principle says no. Is Mach's principle correct? That's the question. --Trovatore (talk) 19:08, 7 June 2010 (UTC)[reply]

Another thing I've wondered about - I walk from A to B on the surface of the globe. I am stationary and the earth is moving. How much energy do I need to rotate the earth? 92.15.24.29 (talk) 20:08, 7 June 2010 (UTC)[reply]

The net energy is zero, since the Earth's rotation is the same at the beginning as the end (stationary, with respect to you). --Tango (talk) 20:20, 7 June 2010 (UTC)[reply]

The fact that I feel tired after walking 20 miles, and the fact that vechiles need fuel to travel, contradicts that assertion. 92.15.14.201 (talk) 23:12, 7 June 2010 (UTC)[reply]

No, it doesn't. You and the car might need to use energy for other reasons different than the one you're thinking, such as replenishing energy lost due to air friction. Dauto (talk) 01:21, 8 June 2010 (UTC)[reply]

Production of phosphine and glow from white phosphorus

I have someone who posted something on my talk page user talk:chemicalinterest#thank you about putting white phosphorus in potassium hydroxide, heating it and watching it glow and evolve what I think is phosphine gas. What is the reducing agent in that reaction (I know the oxidizing agent would be phosphorus)? Water isn't strong enough. --Chemicalinterest (talk) 12:03, 7 June 2010 (UTC)[reply]

Can't they build themselves a simple fume cupboard at least rather than trying to depend on a mask? I'm agin all this nanny codding stopping any experimentation but it is a good idea not to kill oneself. Dmcq (talk) 12:51, 7 June 2010 (UTC)[reply]

If you read phosphine, you'll see that sodium hypophosphite (Na[H2PO2]) and sodium phosphite (Na2[HPO3]) are by-products.

This implies that phosphorus is both the oxidising agent and the reducing agent, i.e. it disproportionates.

Ben (talk) 14:05, 7 June 2010 (UTC)[reply]

So if its disproportionation, why the glow? Does phosphorus glow during disproportionation? Or does it have nothing to do with the disproportionation? --Chemicalinterest (talk) 14:33, 7 June 2010 (UTC)[reply]
Sounds like an example of chemiluminescence, something mentioned in the "Glow from white phosphorus" section of the Phosphorus article. DMacks (talk) 15:02, 7 June 2010 (UTC)[reply]


Sorry, here I am referring to a response is to use White Phosphorus [P] reacts with Potassium Hydroxide solution [KOH] (concentration) and heating this mixture at high temperature, then you will get a Hydrogen gas like sulfur [H2S] Phosphorus is Hydrogen [H3P]. itself highly flammable substance and characteristic fish smell, its own fire in the air with a characteristic blue --I love chemistry (talk) 15:03, 7 June 2010 (UTC)[reply]

So it is the pyrophoricity of the phosphine that makes it glow; not the chemiluminescence of the phosphorus. Now its clear. --Chemicalinterest (talk) 17:49, 7 June 2010 (UTC)[reply]

Why can't we get energy for free from a magnet?

Why wouldn't this work [32] —Preceding unsigned comment added by 90.7.162.110 (talk) 13:26, 7 June 2010 (UTC)[reply]

Because for every reaction there is an equal and opposite reaction. The magnet pulls the car forward, but also pulls the bar that the magnet is attached to backward with the exact same force. You can try it at home! Attach a magnet to your chest, then hold another magnet at arms length, and see if you go anywhere.-RunningOnBrains(talk) 14:18, 7 June 2010 (UTC)[reply]
I sometimes do this ^ while going for a jog, pantomiming being dragged and making a worried face, just to confuse people. SamuelRiv (talk) 15:52, 7 June 2010 (UTC)[reply]
(I have a rule: Everytime someone makes this error, they have to donate a dollar to Wikipedia! My plan is to make a million dollars that way...and I think I'm getting close!)
The problem is that you are confusing "force" with "energy".
A magnet exerts a force - it does not "produce energy". Your fridge magnet will stay stuck to your fridge until hell freezes over without "running down" because it doesn't need to expend energy to do it. Energy is produced or consumed when something that's exerting a force moves. So when you peel the magnet off of your fridge, it costs your muscles energy to do it. When you release the magnet from a half inch away from the fridge, it releases that energy by moving back onto the fridge - and the energy you added is turned into heat and sound as it snaps back onto the metal. The magnet itself doesn't HAVE energy to give away - it's not like a battery or a wound-up clockwork toy. The situation is exactly the same as with another common force...gravity. If you place a book onto a table, the book and the table are exerting forces on each other - the book is being pulled downwards by gravity - and the table has interatomic forces that resist motion and press upwards onto the book to prevent it from moving. Again - there are lots of forces being applied here - but no energy is being produced or consumed in the process because nothing is moving (technically: because nothing is accelerating). The book will stay there, resting on the table forever without the table or the book "running out of energy" because they are doing that. But if you exert chemical energy from your muscles to pick up the book - you are giving it energy because you are moving it against the force of gravity...and when you drop it onto the floor, this energy will be released as it falls, turning into kinetic energy (motion) and then into heat and sound as it hits the floor. So just as we can't get "free" energy from gravity - so we also can't get it from magnets.
SteveBaker (talk) 18:52, 7 June 2010 (UTC)[reply]
Oops. I had better explain that the word "energy" here comes from me -- the OP had titled this section "Question", which I thought needed to be changed. The title as it is now seemed to capture the spirit of the question, but it didn't occur to me that the OP might be blamed for using a word that the OP didn't use. Sorry, Looie496 (talk) 20:26, 7 June 2010 (UTC)[reply]

What are corn syrup solids?

I see this frequently in food ingredients, but the entry on Corn Syrup doesn't seem to mention solids specifically. What are they and why are they used as opposed to liquid corn syrup? --70.167.58.6 (talk) 16:21, 7 June 2010 (UTC)[reply]

My guess is that they are "dehydrated" or crystallized corn syrup. --Rajah (talk) 16:50, 7 June 2010 (UTC)[reply]
As a side question: "Does anyone know of an online source that defines what ingredients on food products in the USA are?" It seems like the FDA should have a website giving precise definitions on some of the vaguer ingredients. Almost like a MSDS for food, if you will. --Rajah (talk) 16:50, 7 June 2010 (UTC)[reply]
They don't want to say "mostly sugars." 67.243.7.245 (talk) 17:53, 7 June 2010 (UTC)[reply]
Right. My favorite new one is cane juice crystals for ordinary sugar. --Trovatore (talk) 20:14, 7 June 2010 (UTC)[reply]
Here's a Everything Added to Food in the United States (EAFUS) database http://www.fda.gov/Food/FoodIngredientsPackaging/ucm115326.htm and a sample search for "corn" http://www.accessdata.fda.gov/scripts/fcn/fcnNavigation.cfm?filter=corn&sortColumn=&rpt=eafusListing

--Rajah (talk) 00:50, 8 June 2010 (UTC)[reply]

Straight lines in Nature

salt
galena

Why there are no straight lines in a perfect system like our nature? —Preceding unsigned comment added by 113.199.209.61 (talk) 16:30, 7 June 2010 (UTC)[reply]

I have never heard any scientific view that nature is perfect. And for straight lines, have you ever looked really closely at a grain of salt? Googlemeister (talk) 16:45, 7 June 2010 (UTC)[reply]
There are straight lines in crystals, I put some images in that show crystals with straight or very nearly straight edges
What about crystals and their Cleavage planes? Aren't they 'straight'?. See also this website [33]220.101.28.25 (talk) Contribs 16:50, 7 June 2010 (UTC)[reply]
This question may be based on a very old (and easily debunked) theory that coastal cities had more advances in art and science than inland cities because the only straight line in nature is the horizon and the only place you can see it is looking out across the ocean. Of course, that isn't the only straight line in nature. It isn't the only place to see an unobstructed horizon. And, there is no correlation between being able to see a straight line and being more advanced in art and science. -- kainaw 16:53, 7 June 2010 (UTC)[reply]
Crepuscular rays (in fact all rays) are straight.--Shantavira|feed me 17:18, 7 June 2010 (UTC)[reply]
The horizon is curved - not straight (and even at sea: waves, tides and gravitational anomalies disturb that "perfect" arc - as does the fact that the earth isn't perfectly spherical). Crepuscular (and in fact all rays) that pass through air (refraction) or a gravitational field are bent by that passage - and there is nowhere in the universe where there are no gravitational fields. Those crystals aren't perfectly straight if you zoom into them enough (see third picture to the right here). So all of those examples are busted.
Zoom into the highest res version of this photo of a salt crystal...the edges are not straight!
The truth is that if you are picky enough, then outside of pure mathematics, there is no such thing as a "perfect" anything. No material substance can be used to make a perfect straight line because of quantum effects - I suspect that the uncertainty principle guarantees that you can't be sure that even an idealized light beam in a gravitation-free universe would travel in a straight line because you can't nail down the positions of the photons that accurately...but we don't live in a gravitation-free universe - so light rays are bent all the time (albeit not by very much). The standard of "perfection" is just too high a bar for anything outside of theory.
However, if you apply a more realistic view of "perfect", there are things like crystals, light rays and many other things that result in straight lines that are close enough to perfect to satisfy any reasonable person.
What is intensely annoying about this statement that "nature" doesn't produce straight lines is that humans can't do any better...even by "unnatural" processes (whatever that means!). There is simply no such thing as perfection in anything that you can measure. The universe simply doesn't work that way. But "nature" (whatever that is) is as capable of approximating a straight line as anything else that's real. So the statement is really a tautology.
SteveBaker (talk) 18:31, 7 June 2010 (UTC)[reply]
Shorter lines would more likely be straight than longer lines. Bus stop (talk) 18:37, 7 June 2010 (UTC)[reply]
Not exactly. Shorter lines typically vary from perfection to a lesser degree than longer ones - but the probability of exact perfection is still zero...so "more likely" isn't a very good description! SteveBaker (talk) 19:13, 7 June 2010 (UTC)[reply]
I may be wrong about this, but since antineutrinos rarely interact with anything (I know they do interact occasionally), wouldn't they travel in straight lines? Or would they still be curved by gravity? Regards, --—Cyclonenim | Chat  23:16, 7 June 2010 (UTC)[reply]
There are plenty of mathematical properties that would usually apply to anthopogenic settings but still occur in nature, such as the Fibonacci spiral, the Golden ratio and fractal patterns. ~AH1(TCU) 00:09, 8 June 2010 (UTC)[reply]
Anti-neutrinos travel in straight lines in spacetime, which would mean they would necessarily be "warped" by gravity (like photons). Whether this constitutes "straight" or not depends on whether you are trying to force Euclidean geometry on inherently non-Euclidean space. --Mr.98 (talk) 00:32, 8 June 2010 (UTC)[reply]
I think Steve is being way over pedantic, of course we aren't asking for some sort of platonic version of a straight line, we're looking for a pretty good approximation of two points joined by a line, not obviously bent or crooked.. Those salt crystals look pretty straight to me even at highest magnification. And if they're not, it could be due to lens aberration. Another example of straight lines in nature is spiders can form straight lines with their webs,
straight lines a plenty.
. Vespine (talk) 03:35, 8 June 2010 (UTC)[reply]

How could moth sleep in day?

This question was on a matchbox- its really interesting one
If moths are attracted to bright light then how come they sleep during day? —Preceding unsigned comment added by Myownid420 (talkcontribs) 16:43, 7 June 2010 (UTC)[reply]
The first thing to consider is no one is sure why moths are attracted to lights. See Moth#Attraction to light. The second thing is that whatever the reasons, there's a fair chance it wouldn't work if they are active during the day since the light is so bright then that there's nothing to be attracted to. Nil Einne (talk) 17:03, 7 June 2010 (UTC)[reply]
I'm attracted to all sorts of things...but not while I'm asleep. Why would moths be any different in that regard? SteveBaker (talk) 18:19, 7 June 2010 (UTC)[reply]
I think I may have misunderstood the question since I didn't pay much attention to the header. I thought the OP was asking why moths sleep during the day if they are attracted to light not how come they can sleep during the day Nil Einne (talk) 21:41, 7 June 2010 (UTC)[reply]
AFAIK, nocturnal moths are not attracted to light; on the contrary, if you wake one during the day it will fly straight to the nearest shade and go back to "sleep". Moths do use the moonlight to navigate, so the streetlights simply throw their navigation equipment off. Circadian rhythms and insect navigation are both fascinating subjects, and, as you can see by the color of the links, the prior is better understood than the latter :) --Dr Dima (talk) 20:40, 7 June 2010 (UTC)[reply]

Thanks Nil u really understood the question correctly. but not at first. simply the question is why not moths go flying towards sun in the morning? as per the first article suggested it could be becoz they want to hide from predators in light, then why not they should hide from them in night. going near light will make them a vulnerable prey.--Myownid420 (talk) 05:03, 8 June 2010 (UTC)[reply]

Columbia nuclear fission vs. Chicago Pile-1

In nuclear fission, "It was clear to a number of scientists at Columbia that they should try to detect the energy released in the nuclear fission of uranium from neutron bombardment. On 25 January 1939, a Columbia University team conducted the first nuclear fission experiment in the United States,[18] which was done in the basement of Pupin Hall; the members of the team were Herbert L. Anderson, Eugene T. Booth, John R. Dunning, Enrico Fermi, G. Norris Glasoe, and Francis G. Slack. The next day, the Fifth Washington Conference on Theoretical Physics began in Washington, D.C. under the joint auspices of the George Washington University and the Carnegie Institution of Washington. There, the news on nuclear fission was spread even further, which fostered many more experimental demonstrations.[19]"

In Chicago Pile-1, "Chicago Pile-1 (CP-1) was the world's first artificial nuclear reactor.[4] CP-1 was built on a rackets court, under the abandoned west stands of the original Alonzo Stagg Field stadium, at the University of Chicago. The first artificial, self-sustaining, nuclear chain reaction was initiated within CP-1, on December 2, 1942."

So, my question is Was the Columbia experiment not artificial, not self-sustaining, or not a chain reaction? or some combination of those? I ask because it seems like to fission uranium, there would have to be a chain reaction, but I"m not clear on the self-sustaining part.

Thank you. --Rajah (talk) 16:47, 7 June 2010 (UTC)[reply]

Columbia was not self-sustaining. It was just a small experiment to detect energy released from fission. CP-1 was a real reactor, where the fission reaction is critical (self-replenishing). --Mr.98 (talk) 17:59, 7 June 2010 (UTC)[reply]
Just to elaborate a bit more: the Columbia experiment involved coating an ionization chamber with uranium oxide, and then putting a neutron source of known intensity inside of it, and then measured the energy set off by the fission of U-235 (in the form of recoil of the fission fragments). This provided direct experimental evidence confirming both the Hahn-Meitner observations and explanation of fission, as well as the Bohr elaborations (in particular, the U-235 was responsible for fission, not U-238). This is basically a desk-top experiment, just showing that the theory of fission seemed to be true. It was not self-sustaining at all—fission happened only when the neutron source was applied, and stopped immediately as it was removed. There was no critical mass. CP-1, by contrast, was a room-sized apparatus of uranium oxide embedded in blocks of graphite, arranged in a critical mass, with the graphite serving as a moderator to slow down the resultant neutrons so that they could more easily be used for future reactions. It went critical—meaning it could have kept exponentially reacting indefinitely (until it melted itself into the floor) had they not stopped the reaction after a few minutes. Each fission reaction produced more than one secondary fission reactions on average. Does that clarify the differences and the significance of each? The Columbia thing was important but not world-changing—it just confirmed what was already expected to happen, and gave some more precise understanding of it. The Chicago experiment was a bigger deal—it proved that reactors could be built. --Mr.98 (talk) 21:20, 7 June 2010 (UTC)[reply]
Cool, thanks. YOu might want to add some of those Columbia details to the nuclear fission article. --Rajah (talk) 00:44, 8 June 2010 (UTC)[reply]

whitening armpits

don't want any medical advice ... just casual ... how can i whiten armpits? —Preceding unsigned comment added by KateGTG (talkcontribs) 18:02, 7 June 2010 (UTC)[reply]

Skin whitening? SteveBaker (talk) 18:15, 7 June 2010 (UTC)[reply]

Mystery not-quite-electrical component

My bulkhead light came with a packet of fittings and rubber seals, but also included were two of these. Can anyone identify this? It looks like some sort of sleeving for wiring but what is it's purpose? --78.148.138.14 (talk) 18:44, 7 June 2010 (UTC)[reply]

yes it's sleeve - I'd imagine if you install the light outside the supply will come through a wall - the cable is probably plastic or rubber so a sleeve would prevent any abrasion on the wire when it passes through rough brick or stone.
Also is it possible that the sleeves are heat resistance (woven glass?) and are intended for use on the inside of the light fitting were it may get warm - I'm clutching at straws.
Also where the cable exits the bulkhead light though a conduit -if it's bent or curved- may help the cable pass through (reduced friction)77.86.124.76 (talk) 19:48, 7 June 2010 (UTC)[reply]
(ec)As you say, sleeving, but I would (also) suggest it is some sort of 'high' temperature sleeving. I have seen (wp:OR) similar come with halogen downlights. It may be to protect wiring from the heat of the lamp? --220.101 (talk) Contribs 19:56, 7 June 2010 (UTC)[reply]
Yes, heat-resistant sleeving. If the plastic insulation on normal wiring gets hot, it can melt and short either to earth or to the other wire. This sleeving protects against this eventuality by maintaining insulation even at high temperatures. Dbfirs 20:52, 7 June 2010 (UTC)[reply]

Unable to focus camera beyond certain zoom

I expect that this only happens in certain situations, but when I took this photo, the autofocus only operated up to a certain zoom, after which it could not focus anymore. Switching to manual focus, I seemed to reach the maximum in either direction without focusing the image. The camera is a Panasonic HDC SD60 but I'm not sure this is relevant. Should I have opened or closed the iris? --78.148.138.14 (talk) 18:49, 7 June 2010 (UTC)[reply]

You really need a special macro lens for doing this kind of super-close-up work. Some lenses have a special "macro" setting - a switch or slider somewhere. SteveBaker (talk) 19:09, 7 June 2010 (UTC)[reply]
Aye, it has a macro setting. There was a picture of a flower than flashed when I zoomed in close enough. Unfortunately, I don't believe that my camera was designed to accept alternative lenses. The camera was about 30-40 cm from the article and then zoomed as far as it could maintain focus. I would like to understand what was limiting its capacity to focus, but if it involves 30 different lenses and prisms jostling around, I should probably just forget about it :( --78.148.138.14 (talk) 19:14, 7 June 2010 (UTC)[reply]
try zooming out and moving closer - see answer below.87.102.17.246 (talk) 20:21, 7 June 2010 (UTC)[reply]
That's normal. Just like your eyes (and for the same reason, roughly speaking) a camera cannot focus closer than a certain distance because there is a limit to the maximum power of the compound lens (if you have two lenses, the power increases as they move closer together, and obviously you can't get closer together than touching, so that is the maximum). Changing the zoom changes that distance. I don't think there is anything you can do about it other than change the lens (if the camera has changeable lenses - I haven't looked up that model). See Lens_(optics)#Compound_lenses for some more information. --Tango (talk) 19:17, 7 June 2010 (UTC)[reply]
I should clarify - your camera may well have more than two lenses, but the principle is the same (the maths is just more complicated). --Tango (talk) 19:19, 7 June 2010 (UTC)[reply]
According to the manual [34] (p.39) macro mode can focus to distances down to 1-4cm (wide angle) or 70cm (telephoto) (it has a zoom lens) - did you try adjusting the zoom?87.102.17.246 (talk) —Preceding undated comment added 20:19, 7 June 2010 (UTC).[reply]
and select "intelligent macro mode" whatever that is.
I was looking for info and found this youtube video of your video camera/camera in macro mode [35] really nice video - the worms are fantastic - nice camera - I recommend you all click on it if you're ok with creepy-crawlies.87.102.17.246 (talk) 20:25, 7 June 2010 (UTC)[reply]
If you get really desperate you can stick a magnifying glass between the lens and subject.. does work..87.102.17.246 (talk) 20:40, 7 June 2010 (UTC)[reply]

why don't tertiary amines react with acyl chlorides?

Et3N seems a worse leaving group than Cl-. After all Et3N is a nucleophilic catalyst but Cl- is not. Or is that R4N+ has repulsion with the partial positive charge on the carbonyl, allowing the Cl- to reattack and displace the tertiary amine? So actually, shouldn't the tertiary amine react, but not stay on for very long? John Riemann Soong (talk) 18:52, 7 June 2010 (UTC)[reply]

They do react - eg
MeCoCl + NMe3 >>> MeC(O)N+Me3 Cl-
It's rare to (try to) isolated these, but simple tertiary amines are used to 'activate' acyl chlorides , and as an intermediate step when the acylation of an acid sensitive (or HCl sensitive) substance is attempted.
I'll see if I can find a literature example77.86.124.76 (talk) —Preceding undated comment added 19:43, 7 June 2010 (UTC).[reply]
Okay because some resources appear to state that tertiary amines "don't react with acyl chlorides" and use them as a means of distinguishing them from secondary and primary amines. John Riemann Soong (talk) 19:46, 7 June 2010 (UTC)[reply]
Yes, they do say that - of course any compound formed won't survive an aqueous work-up. - you'll get the tertiary amine back (as a salt) - so in that sense it is true.
Just to show I'm not making this up [36] - example with benzoyl chloride, not acetyl chloride.87.102.17.246 (talk) 20:01, 7 June 2010 (UTC)[reply]

Pendulum

Okay, for a pendulum to have a period independent of its amplitude, the curve of the pendulum's bob must follow a cycloid. How was Huygens able to change the path that pendulum took? I would imagine that he had curves near the pivot so that as the pendulum swung, it would wrap around it. But what shape would these curves have to be? Would they be cycloids too? 173.179.59.66 (talk) 19:37, 7 June 2010 (UTC)[reply]

Yes. Dauto (talk) 20:05, 7 June 2010 (UTC)[reply]
The period of a circular-arc pendulum is approximately independent of amplitude for small amplitudes. Huygens calculated that the cycloid was the ideal curve, and he had pendulum clocks built, but I don't know whether he bothered with the tiny curve corrections because, in practice, pendulum clocks usually have a fairly constant and small amplitude. Dbfirs 20:22, 7 June 2010 (UTC)[reply]
I know the correction has been used but I don't know if Huygens bothered doing it. Dauto (talk) 20:44, 7 June 2010 (UTC)[reply]
Does it make any difference to accuracy? I would think that stability of the suspension and temperature correction would be much more significant in real clocks. Huygens seems to have been an exceptionally talented scientist so he may well have tested out the design (or got his clockmaker to try it out - he wasn't very practical). If he did, then I suspect that he found that it wasn't worth the trouble. Has anyone access to any of his research papers? Dbfirs 21:02, 7 June 2010 (UTC)[reply]
A spring is used to support the pendulum on some clocks to compensate pretty accurately for this problem. Some early clocks used cycloidal jaws but they were more trouble than they were worth. Dmcq (talk) 22:47, 7 June 2010 (UTC)[reply]
Thanks, though I don't see what "problem" needs compensating for if the amplitude is kept small and almost constant. I would have thought that a perfectly rigid support would be the way to improve accuracy. Dbfirs 00:17, 8 June 2010 (UTC)[reply]

OP: Yeah I know that he ended up abandoning the cycloid thing. My question was more of a mathematical one. Is there an easy way to see that cycloidal jaws maked the bob move in a cycloid? 173.179.59.66 (talk) 03:04, 8 June 2010 (UTC)[reply]

Perhaps I'm imagining this the wrong way, but placing any curve under the pivot would, the way I imagine it, increase the "problem" by causing the bob to move in a tighter curve, not the flatter curve of a cycloid. Allowing the pivot to move sideways would introduce additional inaccuracies greater than those eliminated wouldn't it? Is my intuition wrong, or am I missing something that is obvious to others? Can anyone draw or link a picture? Dbfirs 08:36, 8 June 2010 (UTC)[reply]

Melting points of various ionic solids

I thought that the melting point of an ionic solid would be determined by the difference in electronegativity between the two elements composing the salt. But even though each alkali metal and alkaline earth metal halide follows the rule, some other ones don't. For example, why isn't beryllium oxide low melting because it is more covalent than caesium oxide, which is very ionic? --Chemicalinterest (talk) 20:21, 7 June 2010 (UTC)[reply]

The main factors are:
  • Size (mp increases with decreasing size due to inverse relationship between force and electric charge at distance)
  • Charge - higher charge means higher mp.
[37] -see section "The physical properties of sodium chloride" which repeats what I said above.
Low difference in electronegativity means the compound is more covalent - it doesn't affect melting point that much if the compound is a macromolecule.87.102.17.246 (talk) 20:28, 7 June 2010 (UTC)[reply]
The lattice enthalpy can be a significant contributor to melting point. See Kapustinskii equation and Born-Landé_equation for more info. Of course, these assume that bonding is 100% ionic. Brammers (talk/c) 21:04, 7 June 2010 (UTC)[reply]

Fusion reactors... what the hell.

Fusion is clearly a process that requires phenomenally high temperatures to work. How the hell have scientists managed to, or at least planned to, contain the plasma? I would have thought any conceivable metal unable to withstand such temperatures, but clearly I'm mistaken. Is the plasma prevented from touching the walls of the chamber, and conducted in a vacuum to prevent direct transmission of heat, or are there other components at work? Regards, --—Cyclonenim | Chat  23:20, 7 June 2010 (UTC)[reply]

Yes and yes. Hope you don't mind but I'm just going to paste a template in:
The articles under "magnetic confinement" are the ones you want - tokamak is a common one - the plasma in general doesn't touch anything - magnetic (and electric?) fields are used to keep it away from the sides of the reactor.87.102.17.246 (talk) 23:49, 7 June 2010 (UTC)[reply]
In theory you use a "magnetic bottle"—you ionize the plasma and then contain it with an electromagnetic field, so it doesn't touch anything. In practice, you try very hard to make a stable magnetic bottle, and it inevitably leaks out heat and is very hard to keep stable and the plasma generally leaks out. A plasma physicist friend once described it as trying to push all of the water from one side of your tub to the other by just using your bare hands—the plasma generally finds a way through. This is basically why controlled fusion (that is, outside of H-bombs) has proven so difficult and elusive. The current front-runner of this type is the ITER program, where they are trying to overcome the instabilities of the bottle by just making it huge. Whether it will work or not (to the point of achieving more energy from the fusion reactions than it takes to start the reaction) is not clear (scientists disagree).
The other main approach to fusion, inertial confinement fusion, doesn't use any kind of confinement at all. You basically just try to start up a fusion reaction inside of a big chamber, and then catch the energy that shoots out from it. The trick here is that starting controlled fusion reactions in small spaces in a short amount of time is not easy. The current front-runner of this type, the NIF, uses a set of gignormous lasers to try and ignite fusion reactions in a capsule the size of a pinhead. Whether it will achieve the "break-even" point is also not known, but it seems reasonably likely that it will happen. (Whether either of these will ever prove to be an economical way to generate energy is a separate, and far more problematic, question.) --Mr.98 (talk) 00:16, 8 June 2010 (UTC)[reply]

hard wood floors

why do my hard wood floors turn black if they get wet —Preceding unsigned comment added by Alexsmith44 (talkcontribs) 23:23, 7 June 2010 (UTC)[reply]

Many things get darker when they get wet. Many types of cloth, wood, concrete just to name a few. I don't know what the phenomenon is called. --Chemicalinterest (talk) 00:09, 8 June 2010 (UTC)[reply]

how does water eat thru polyurethane thou? —Preceding unsigned comment added by Alexsmith44 (talkcontribs) 03:32, 8 June 2010 (UTC)[reply]

(relisted below at "how does water eat thru polyurethane thou?"--220.101 (talk) Contribs 04:10, 8 June 2010 (UTC)[reply]


what? —Preceding unsigned comment added by Alexsmith44 (talkcontribs) 04:48, 8 June 2010 (UTC)[reply]

Spaceship without windows

You are in a spaceship without any windows or external instruments. a) You are travelling at say 50% of the speed of light, and are captured into orbit by a massive object such as a star or black hole. Your spaceship traces a sharp curve around it, almost doubling back, and begins to orbit it. Does anything feel different from normal inside the spacecraft? Could you detect that your direction has changed or feel any force from the change in direction? b) You fall into a black hole. Does anything feel different from normal inside the spacecraft, or are you unaware of this happening? 92.28.247.252 (talk) 23:35, 7 June 2010 (UTC)[reply]

You wouldn't feel the gravity directly, but you might feel the tidal forces, particularly from a black hole. Incidentally, you won't usually go into orbit about a massive object just by passing it - you would have to decelerate in order to be captured. In order to end up in a periodic orbit without thrusting, you have to have started out in a periodic orbit. Oh, and you couldn't orbit a star at 50% the speed of light - the orbit an object the mass of the sun at 0.5c you would need to have an orbital radius of about 6km, which is much much smaller than the sun, so is impossible. If you want to orbit something at those kind of speeds, it needs to be a black hole (or possibly a neutron star). --Tango (talk) 23:49, 7 June 2010 (UTC)[reply]
In case you don't know, tidal forces are because the gravity on the side of the spaceship closer to the star is slightly greater than the gravity on the side farther from the star - and you might be able to measure the difference. But otherwise no, you won't be able to tell. And tidal forces are not always very small, see Spaghettification. Ariel. (talk) 06:01, 8 June 2010 (UTC)[reply]

June 8

Sparrows are Lousy Parents

There was a sparrow nest under an air conditioner I was moving, and, feeling benevolent, I relocated the chicks to another location. They are now in a young cherry tree close to the original nest, but the original nest was—if I may criticize another species' instinctual skills—a shoddy piece of garbage, and it fell apart. So, I used an abandoned nest built by, probably another species of bird—a bird with some pride in its craft.

Okay, not too promising, I know. Even less promising—the derelict sparrow parents have proceeded to rebuild their (dirty, probably soon to be disease/mite ridden) nest under the air conditioner—and, I imagine, all of this in lieu of tending to their old kids.

I'm (A) interested in what's going on here; (B) interested in what I can do to maybe spark a family reunion; and (C) if B is not possible, interested in maybe considering a humane way to divert the chicks' slow and painful route to becoming a gross meditation on death...

Thanks. Alfonse Stompanato (talk) 00:15, 8 June 2010 (UTC)[reply]

Whenever you touch baby birds, especially if you move them, there is a high chance of the parents rejecting them (I'm not really sure why - possibly because they aren't sure the chicks are theirs). I don't think there is much you can do now. You can take the chicks to a vet and get them put down, but that's about it (I would just let nature take its course now - it's always possible the parents will return to them, but it seems unlikely - they seem to be planning a second attempt, which suggests they think their first lot of chicks are dead). Really, you should have moved the air conditioner either before the eggs were laid or after the chicks had left the nest. Unfortunately, I think you've killed those chicks. --Tango (talk) 00:57, 8 June 2010 (UTC)[reply]
Actually, the whole "if you touch a baby bird, the mom will reject it" is a bit of an old wives tale. ([38] [39] [40], and others by Googling "touch baby bird") This seems like something we would have an article on. The recommendation *is* to leave them alone, but that's because falling out of the nest is normal. However, if you do touch/move them, you definitely don't have to "put them down". -- 174.24.195.56 (talk) 02:20, 8 June 2010 (UTC)[reply]
And just by the way "sparrows are lousy parents" is a conclusion you only jump to if you unnecessarily anthropomorphize the situation. Their option of efficiently "trying again" is no doubt a great strategy for a species that doesn't spend 9 months gestating their young. They only take 3 or 4 weeks to hatch some young, so loosing one lot of eggs is obviously not as big a set back for a species that can lay and hatch potentially every few months, as opposed to only once a year. I can think of several valid critiques of human breeding strategies from a sparrow's point of view. Vespine (talk) 05:52, 8 June 2010 (UTC)[reply]
Sparrows (like many birds) are position dependent for parenting - they build a nest, and tend their children by returning to that position and feeding the chicks that are there. This is perfectly adequate for most small birds, whose parenting responsibilities might last a handful of weeks until the chicks learn to fly (and it has pros and cons - the parent does not waste time searching for missing chicks which were most likely victims of predators, but are also vulnerable to parasitic birds like cuckoos and cowbirds, who lay their eggs in other birds' nests). Birds generally have poor senses of smell, so they don't smell human scent, but they are attentive to their nests, and will notice if people spend an inordinate amount of time around them (and may not return to a disturbed nest on the assumption that the chicks - again - were victims of predation). --Ludwigs2 06:19, 8 June 2010 (UTC)[reply]
Well, the OP could take the chicks to one of those wildlife rescue places, if they take that species (and are not all like 'meh, there's millions of those - nowt we can do', which does happen) or failing that, attempt to raise the chicks himself (if legal where he is, of course - may not be), which is not easy at all (understatement!) with small, fragile, altricial birds like sparrows. I do agree with Tango that it doesn't look good for these particular nestlings, however. --Kurt Shaped Box (talk) 06:44, 8 June 2010 (UTC)[reply]
According to the Bible, ostriches are cruel parents. (http://multilingualbible.com/job/39-16.htm; context in verses 13–17)
-- Wavelength (talk) 07:00, 8 June 2010 (UTC)[reply]
"Sparrows are Lousy Parents" WTF!? there are millions of them, they must be one of the most successful parents. 86.4.183.90 (talk) 07:45, 8 June 2010 (UTC)[reply]

Are there any chemicals which are 4 in all three areas? --75.25.103.109 (talk) 01:09, 8 June 2010 (UTC)[reply]

The "best" I can find is nitroglycerine at 3-3-4. Nitrogen triiodide seems like a good bet since it's extremely explosive, but I can't find a link showing the NFPA rating for it. 75.157.57.12 (talk) 01:50, 8 June 2010 (UTC)[reply]
tert-Butyl hydroperoxide is 4/4/4 and also "OX" special-hazard for added fun. DMacks (talk) 02:38, 8 June 2010 (UTC)[reply]
I can't find the code for Peroxyacetyl nitrate but there must be some 5's in there..87.102.17.246 (talk) 05:37, 8 June 2010 (UTC)[reply]
Bretherick's Handbook of Reactive Chemical Hazards (aka Boy's big book of blowing buddies to bits) is your friend.87.102.17.246 (talk) 05:54, 8 June 2010 (UTC)[reply]
I think you'll like this blog. In particular the category I linked to. Chlorine trifluoride is still my favorite from that list. Ariel. (talk) 06:04, 8 June 2010 (UTC)[reply]

Bicycles and biomechanics...

Bicycles are famous for being such a sterling example of design through experimentation - it took decades for people to understand what made bicycles work, and how best to design them. And yet, the pedal motion remains a perfect circle whereas whether walking or running, the path traveled by the human foot is elliptical.

Has any research been done on what sort of path (circular, ovoid, etc) is most efficient for the legs? For the sake of the question, let's ignore the obvious design difficulties presented by an elliptical pedaling system. I'm just interested in what using our legs at optimum efficiency looks like... 218.25.32.210 (talk) 02:08, 8 June 2010 (UTC)[reply]

Well, no one has had a go yet so I'll just have a wild guess, the regular circular pedal path might only deviate from the "perfect optimal" path by a few centimeters. Trying to make an efficient elliptical pedaling "system" would probably add weight and complexity that would outweigh (literally and figuratively) the few % improvement you might get from the slightly better pedal action. Vespine (talk) 05:39, 8 June 2010 (UTC)[reply]
A piston action would, I suspect, be slightly more efficient from a human muscle point of view, but the legs deliver power on the downstroke of the circle, so the advantage would be small. I would be surprised if variations on design have not been tried, but perhaps the loss in power in converting the force to drive the chain outweigh the gain from a longer straighter leg-stroke. Can anyone find any research? Dbfirs 08:29, 8 June 2010 (UTC)[reply]
Try search "bicycle crank mechanism" - many results many of which are patents. Here's one example [41] 87.102.17.246 (talk) 08:52, 8 June 2010 (UTC)[reply]
On a similar line elliptic and ovoid drives have been produced eg search "elliptical bicycle crank"
I've no idea if any of these are any good or just 'bright ideas'. These are mostly designs to eliminate dead spots, haven't found a straight stroke mechanism yet. But they definately exist - no problem buying them. 87.102.17.246 (talk) 09:08, 8 June 2010 (UTC)[reply]

how does water eat thru polyurethane thou?

how does water eat thru polyurethane thou? —Preceding unsigned comment added by Alexsmith44 (talkcontribs) 03:32, 8 June 2010 (UTC)[reply]
(Copied from above re "hard wood floors"--220.101 (talk) Contribs 04:04, 8 June 2010 (UTC))[reply]

titanium

how come titanium is sometimes dull and sometimes shiny? like this titanium bowl below is dull but iv seen shiny titanium too. if its just metallic titanium shouldent it only have one finish?

http://www.rei.com/product/720286


--Alexsmith44 (talk) 04:51, 8 June 2010 (UTC)[reply]

That's probably been shot peened to give it a non-polished finish (or another process - it may not have been shot peened)
Same thing goes with steel - you can have polished steel or brushed steel or other finishes.87.102.17.246 (talk) 05:26, 8 June 2010 (UTC)[reply]

color of elements

how does elements got there colors? whats there at atomic level to give them color?What abo't compounds and mixtures? --Myownid420 (talk) 05:15, 8 June 2010 (UTC)[reply]

This has been asked many times in the past, the answer is "it varies". There is no single quick and simple answer. Our article on Color goes into quite a bit of detail, have a read of that and come back if you have any specific questions. Vespine (talk) 05:33, 8 June 2010 (UTC)[reply]